BAR. All Subject MBE Questions (that I got wrong)

¡Supera tus tareas y exámenes ahora con Quizwiz!

An ordinance of a city prohibits leafleting on the grounds of any hospital or on the sidewalks within five feet of the hospital entrances during visiting hours. A member of a religious group advocating prayer to restore the sick to good health is arrested for violating the ordinance, is fined $100, and is convicted. She appeals her conviction, claiming that her constitutional rights were violated.The case was heard by the state supreme court, which ruled that while the ordinance was permissible under the United States Constitution, it was unconstitutional under the state constitution because the fine money was designated to go to the city's only hospital, which was privately owned, rather than to the city. The city seeks to bring the case before the United States Supreme Court.Should the United States Supreme Court grant certiorari? A)No, because the case was decided on independent state grounds. B)No, because the case is moot. C)No, because this is a political question. D)Yes, the Supreme Court should hear the case on its merits, because it involves an important federal question.

A.

A corporation whose subsidiaries include a major hotel chain planned to build a new hotel and advertised for bids to build the hotel within the next six months. Four bids were received, for $17 million, $17.2 million, $17.4 million, and $15 million. The corporation's chief financial officer reviewed the bids, then emphatically told the corporation's chief executive officer ("CEO") that there was "no way" the low bidder could make a profit on the $15 million bid. The CEO made no response. In fact, the builder had stayed up for 72 hours without sleep preparing the bid for the hotel project and had neglected to include the plumbing expenses in the bid. Typically, the cost of plumbing, including the shop's profit, would have been about $2 million. Shortly after the $15 million contract was signed by the CEO and the builder, the builder discovered his mistake and telephoned the CEO to tell her that he had forgotten to include the cost of plumbing, adding that he would normally charge $2 million for plumbing. The CEO agreed to pay the additional $2 million, but this arrangement was never reduced to writing. After the builder completed the project on time, the CEO sent him a check for only $15 million. Can the builder compel the CEO to tender the additional $2 million? A. Yes, because the CEO was on notice of the builder's mistake. Incorrect B. Yes, because the builder relied to his detriment on the CEO's promise. C. No, because the builder had a preexisting legal duty to complete the project for $15 million. D. No, because evidence of the agreement to pay the additional $2 million is barred by the parol evidence rule.

A. The builder will be able to compel the CEO to pay the additional $2 million because the CEO was on notice of the mistake. The builder has the defense of unilateral mistake. Although the general rule is that a contract will not be avoided by a unilateral mistake, there is an exception where the nonmistaken party either knew or should have known of the mistake. Here the facts clearly indicate that the CEO knew that the builder's bid could not be correct, yet relied on it anyway. Thus, the builder had grounds to avoid the contract. Rather than completely avoid the contract here, the parties agreed to reform it, but they failed to record the reformation in writing. Nevertheless, the court will allow the parties to show the reformed terms because of the mistake. (B) is incorrect because the fact that the builder relied to his detriment on the CEO's promise would not allow him to collect the additional $2 million. Detrimental reliance is a factor in promissory estoppel, which is a doctrine used by the courts to avoid an unjust outcome when there is no contract. Here, there is a contract supported by consideration, so this doctrine does not apply. The builder is entitled to reformation of the bargained-for contract because the CEO was on notice of the builder's mistake. In addition, even if this were a proper case for promissory estoppel, the builder would not necessarily be entitled to the $2 million. Courts that follow the Second Restatement approach typically award reliance, not expectation, damages. (C) is incorrect because the unilateral mistake here was sufficient to discharge the builder from his duties under the contract, so there was no preexisting duty. (If the mistake had not been sufficient to discharge the builder, (C) would be correct because where one is under a preexisting legal duty to perform, performance of that same duty generally will not be sufficient consideration to support a promise to pay additional sums for the performance.) (D) is incorrect because the parol evidence rule only prevents introduction of oral statements made prior to or contemporaneously with a written contract. Here, the $2 million term, although oral, was agreed upon after the original contract was made; thus, the parol evidence rule would not be a bar.

A city council passed an ordinance providing: "No person may contribute more than $100 annually to any group organized for the specific purpose of supporting or opposing referenda to be voted on by the city electorate or regularly engaging in such activities." If the ordinance is challenged in federal court, how should the court rule on the constitutionality of this ordinance? A. Strike it down, because it violates First Amendment rights of free speech and freedom of association. B. Strike it down as a violation of due process, because no hearing mechanism has been provided for. C. Uphold it, because the city council has a legitimate interest in controlling such contributions. D. Dismiss the case, because it involves a political question and is thus a nonjusticiable matter.

A. The federal court should strike the ordinance for violating the First Amendment. While the government may limit the amount of contributions that an individual can contribute to a candidate's campaign (to avoid corruption or the appearance of corruption), the government may not limit the contributions to a political committee that supports or opposes a ballot referendum, because such a law does not serve a sufficiently important interest to outweigh the restraints that it puts on the First Amendment freedoms of speech and association. (B) is incorrect because the Due Process Clause does not require that every law provide for a hearing, but rather only those laws involving the deprivation of life, liberty, or property of an individual. The law here does not involve a deprivation of life or property, and liberty is not being denied to individuals on a judicial basis (i.e., according to the facts of each case), but rather is being denied to all persons on a legislative basis. In such a case, individual hearings are not required to satisfy due process; as long as the law was lawfully adopted (e.g., with notice to all interested parties), the Due Process Clause has been satisfied. (C) is incorrect because a legitimate interest in controlling contributions to a political committee for ballot referendum is not enough. The statute must be "closely drawn" to match a "sufficiently important interest," which is an intermediate scrutiny standard, and the Supreme Court has invalidated limitations on contributions to influence referendum elections. (D) is incorrect because political questions, which are nonjusticiable, arise when the issue is committed to another branch of the government by the Constitution or is incapable of resolution and enforcement by the judiciary. Determining whether a law is valid is within the realm of the judiciary and certainly is capable of resolution (i.e., the law could be invalidated). Thus, there is no political question here.

A plaintiff read of the success of a box-office hit movie about aardvarks in various entertainment journals. The movie was enormously popular among young children, and cartoon figures from the movie began appearing on T-shirts, soft drink mugs, and other novelties. The plaintiff filed suit against the studio alleging that the production company unlawfully used his ideas for the movie. The studio admitted that it had received a clay model of a cartoon animal from the plaintiff, but denied that the model had any substantial similarity to the now-famous aardvarks. The studio had returned the model to the plaintiff, but he had destroyed it. For the plaintiff to testify at trial as to the appearance of the model, which of the following is true? A. The plaintiff can testify as to the appearance of the model because he has personal knowledge of it. B. The plaintiff must show that the destruction of the model was not committed in bad faith. C. The plaintiff must introduce a photograph of the model, if one exists. D. The plaintiff must give advance notice to the opposing party that he plans to use such oral testimony in his case.

A. The plaintiff can testify as to the appearance of the model because he has personal knowledge of it. A witness must be competent to testify, which includes the requirement that he have personal knowledge of the matter he is to testify about. Here, the plaintiff has personal knowledge of the model, as he is the person that had submitted it to the studio. Thus, he is competent to testify as to the model's appearance. (B) is wrong because it states the foundation requirement for the admissibility of secondary evidence under the best evidence rule (also called the original document rule), which does not apply under these circumstances. The best evidence rule covers writings and recordings, which are defined as "letters, words, numbers, or their equivalent, set down in any form." A clay model clearly does not fit within that definition. Similarly, (C) states an acceptable form of secondary evidence under the best evidence rule, which does not apply here. Note, however, that under the Federal Rules (unlike most states), there are no degrees of secondary evidence. Therefore, this choice would be wrong even if the best evidence rule were applicable, because the plaintiff would not be limited to photographic evidence. (D) is wrong because it incorrectly assumes that notice must be given. This type of notice is not a prerequisite for the plaintiff's testimony even had the best evidence rule been applicable.

A pedestrian was injured in an auto accident caused by a driver. The pedestrian's injuries included a broken nose and a broken toe. Not sure of the strength of her case, she sued the driver only for the injuries to her nose. She was awarded $15,000 in damages. Encouraged by this success, she now wishes to sue the driver for the injuries to her toe. May she sue the driver again? A. No, because all related claims "merged" with the final decision in the first case. B. No, because she will be collaterally estopped from pursuing the claim. C. Yes, and she may use the prior decision against the driver. D. Yes, but she will have to relitigate the driver's liability.

A. No, because all related claims "merged" with the final decision in the first case. The pedestrian may not sue the driver again because of merger. Merger occurs when the plaintiff wins; her cause of action is said to "merge" into the judgment such that she cannot relitigate the cause of action later. The court would hold that all of the personal injuries received by the pedestrian in one accident constitute a single cause of action, and that claim preclusion principles forbid relitigation. (B) is incorrect. Issue preclusion, also called "collateral estoppel," applies to issues, not entire cases. (C) and (D) are incorrect for the reasons stated above.

Three drivers were in an automobile accident in a city in State A. The drivers were citizens of State A, State B, and State C. The State B driver filed a tort action against the other two in a State A state court, seeking $300,000 for her severe injuries. The State C driver wants to remove the action to a federal district court. Is the action removable? A. No, because one of the defendants is a citizen of State A. B. No, because an action may be removed from state court only if it "arises under" federal law. C. Yes, because there is complete diversity of citizenship and the amount in controversy exceeds $75,000. D. Yes, because one of the defendants is a citizen of a state other than State A.

A. No, because one of the defendants is a citizen of State A. The action is not removable. Under 28 U.S.C. section 1441, a defendant may remove an action that could have originally been brought in the federal courts. (In other words, subject matter jurisdiction based on either a federal question being presented or on diversity of citizenship would have been present had the case been filed in federal court.) However, a case may not be removed on the basis of diversity jurisdiction if a defendant is a citizen of the state in which the action was filed. Here, the action was filed in State A against a State A defendant; thus, the case may not be removed, even though it is a State C defendant who is seeking removal. This fact also makes (D) incorrect. (B) is incorrect because a case may be removed based on diversity, with the restriction that removal is not available if one of the defendants is a citizen of the forum state. (C) is incorrect. Even though the case satisfies the requirements of diversity jurisdiction, the "in-state defendant" restriction prevents removal.

A woman sued her ex-husband for installments due under their divorce property settlement agreement. The ex-husband defended the suit on the ground that the ex-wife was in breach of the agreement. However, the court granted judgment for the ex-wife. Now, the ex-wife is suing her ex-husband for other installments due under the agreement. The ex-husband raises the defense that the agreement was void and illegal because of fraud perpetrated by his ex-wife at the time the agreement was signed. The ex-wife moves to strike her ex-husband's defense, claiming issue preclusion. Should the court grant the motion? A. No, because the issue was never litigated. B. No, because judgment on one installment does not bar subsequent actions on other installments. C. Yes, because the husband should have raised the defense in the first action. D. Yes, because the issue of fraud is never waived.

A. No, because the issue was never litigated. The court should not grant the motion. Generally, issues actually litigated between the parties are binding on them in subsequent actions concerning the same claim. If the second suit involves a different claim, the first judgment may be invoked as to all matters actually litigated and determined in the first action, provided that the findings were essential to the first judgment. Here, the fraud issue was not actually litigated in the first action, and thus it can be raised with respect to the later installments. (B) is incorrect. The general principle recited in this answer is true for claim preclusion (unless there is an acceleration clause). However, even if claim preclusion does not apply, if the defendant raises the same issues as in the first case, he may be prevented by issue preclusion (collateral estoppel) from re-litigating those issues in that subsequent case. (C) is incorrect. Claim preclusion (res judicata) would bar claims arising out of the same transaction or occurrence. Here, claim preclusion does not apply because a defense, not a claim, is involved, and the second installment is a separate transaction or occurrence. (D) is incorrect because there is no such rule.

The Privileges or Immunities Clause prohibits states from infringing upon their citizens': A. Right to travel B. Right to earn a living C. Right to an attorney D. Freedom of speech

A. Right to Travel

A pedestrian walking on the sidewalk was struck by a car backing out of a driveway. The driver did not see the pedestrian because her neighbor's bushes obscured her view of the sidewalk. The pedestrian was seriously injured and brought suit against the driver and the neighbor. The pedestrian also included the city in his lawsuit, alleging that the city failed to enforce its ordinance requiring homeowners to provide a clear view of sidewalks where they intersect with driveways. The trier of fact determined that the driver was 60% at fault, the neighbor was 30% at fault, and the city was 10% at fault. The jurisdiction has adopted comparative contribution in cases applying joint and several liability. Which of the following is a correct statement regarding liability? A. The city is liable to the pedestrian for the full amount of the damage award. B. Both the driver and the neighbor are liable to the pedestrian for 90% of the damage award. C. Each of the three defendants are liable to the pedestrian for one-third of the damage award. D. The driver is liable to the pedestrian for 60% of the damage award, the neighbor is liable for 30% of the damage award, and the city is liable for 10% of the damage award.

A. The city is liable to the pedestrian for the full amount of the damage award.

A dairy farm operated a small processing plant that supplied premium ice cream to nearby specialty shops and ice cream parlors. It entered into a written agreement with a local ice cream parlor to sell "all output" of its Extra Rich ice cream to the ice cream parlor, and the ice cream parlor agreed to sell exclusively the dairy farm's Extra Rich frozen desserts. The agreement stated that the ice cream parlor would pay $25 for each five-gallon container of Extra Rich ice cream that it ordered from the dairy farm. Several months after the parties entered into this contract, demand for high-fat ice creams dropped sharply among the health-conscious consumers who had formerly patronized the ice cream parlor, and the proprietor had to throw out some of its product because the reduced demand meant that opened containers were not used up before the taste of the ice cream became affected. The ice cream parlor wanted to stop selling the dairy farm's Extra Rich ice cream and instead sell a frozen yogurt product produced by another dairy. Can the dairy farm enforce its agreement against the ice cream parlor? A. Yes, because changing demand is one of the standard risks of business that both parties assumed. B. Yes, because the court will imply a promise on the part of the ice cream parlor to use its best efforts to sell the dairy farm's Extra Rich ice cream. C. No, because there was no consideration on the part of the ice cream parlor to support an enforceable contract. D. No, because the total price and total quantity terms were never established.

A. Yes, because changing demand is one of the standard risks of business that both parties assumed. The ice cream parlor has no grounds for avoiding its obligations under the contract with the dairy farm. In effect, the ice cream parlor is advancing the position that its duty to perform under the contract is discharged by impracticability. In contracts for the sale of goods under the UCC, a party's duty to perform may be discharged where performance would be impracticable. Impracticability exists where a party encounters extreme and unreasonable difficulty and/or expense, and such difficulty was not anticipated. Duties will not be discharged where performance is merely more difficult or expensive than anticipated. The facts giving rise to impracticability must be such that their nonoccurrence was a basic assumption on which the contract was made. Where, as here, parties enter into a contract for the sale of goods to be supplied to the public through a retail outlet, both parties must anticipate the possibility that there will be a change in market conditions, resulting in either an increased or decreased demand for the product. Although the decreased demand results in increased expense to the ice cream parlor in performing its contract because of waste, such difficulties arising from changing demand are to be anticipated. Thus, the ice cream parlor does not have the right to no longer buy any of the dairy farm's Extra Rich ice cream. Note that under the UCC, a shutdown by a requirements buyer for lack of orders may be permissible if the buyer is acting in good faith [UCC §2-306, comment 2], but this right would arise only if there were no longer a market for frozen desserts entirely, and that is not the case here. Here, the ice cream parlor simply wants to curtail its losses by selling a more popular type of frozen dessert, which is forbidden by the exclusivity provision. Thus, the ice cream parlor continues to be bound by its duties under the agreement with the dairy farm. (B) is incorrect because, although a court will imply a promise on the part of the ice cream parlor to use its best efforts to sell the dairy farm's products, the facts do not indicate that the ice cream parlor did not use its best efforts. At issue here is whether, despite those efforts, circumstances exist that were unanticipated and now create extreme and unreasonable difficulty or expense for the ice cream parlor in the performance of its contractual duties. (C) is incorrect because the ice cream parlor relinquished its legal right to sell any frozen desserts other than those of the dairy farm. This giving up of a legal right constitutes legal detriment to the ice cream parlor, so there is consideration. (D) is incorrect because an agreement to buy or sell all of one's requirements or output is capable of being made certain by reference to objective, extrinsic facts (i.e., the buyer's actual requirements or the seller's actual output). There is an assumption that the parties will act in good faith; thus, there may not be a tender or demand for a quantity unreasonably disproportionate to a stated estimate or prior output or requirements. Here, the agreement by the dairy farm to sell all of its output of Extra Rich ice cream to the ice cream parlor can be made certain by referring to such factors as the normal output of such product by the dairy farm. In addition, the ice cream parlor agreed to pay $25 for each container of Extra Rich ice cream, so the total price is also ascertainable. Thus, it is not a ground for avoiding enforcement that the total price and quantity were not established.

A customer choked on a bone in her fish fillet while eating at a restaurant. The restaurant's manager, who was not in the restaurant when the incident occurred, met with the customer and the restaurant employees who witnessed the incident. As required by state law, the manager prepared a report regarding the incident and filed it with the state board of health. The manager then accepted a job in another country and moved there. The customer subsequently commenced a civil action against the restaurant in a federal court, alleging negligence and a breach of the warranty of merchantability. The customer's attorney served a request for all documents and reports prepared by the restaurant relating to the incident. Must the restaurant produce the manager's report? A. Yes, because the manager prepared the report to comply with state law rather than to prepare for trial. B. Yes, because the customer can establish that she has a substantial need for the report to prepare her case. C. No, because the manager's report is hearsay and therefore not discoverable. D. No, because the report constitutes work product and is therefore not discoverable.

A. Yes, because the manager prepared the report to comply with state law rather than to prepare for trial. The restaurant must produce the report because it is relevant, proportional to the needs of the case, and it was not prepared in anticipation of litigation. Generally, a party may obtain discovery regarding any matter that is proportional to the needs of the case and that is relevant to any party's claim or defense, provided the matter is not subject to a privilege or to the exception for trial preparation materials, also called work product. Trial preparation materials are documents or other tangible things prepared in anticipation of litigation or for trial. Here, the manager's report is relevant because it contains facts relating to the incident that is the basis of plaintiff's claim. Moreover, it is not subject to the exception for trial preparation materials because it was prepared in the ordinary course of business to comply with state law, not in anticipation of litigation. Finally, the costs of producing the report likely would not be prohibitive. (B) is incorrect because it invokes the exception to the rule against discovery of work product, namely that work product is discoverable if the party seeking it "has substantial need for the materials to prepare its case and cannot, without undue hardship, obtain their substantial equivalent by other means." This exception is not pertinent because the manager's report is not work product. (C) is incorrect because material need not be admissible as evidence to be discoverable. The standard is relevancy, not admissibility. (D) is incorrect because, as discussed above, the report is not work product.QUESTION ID: MJ153

A small processor of specialized steel agreed in writing with a small manufacturer of children's toys that it would supply, and the manufacturer would buy, all of the manufacturer's specialized steel requirements over a period of years at a set price per ton of steel. Their contract did not include a nonassignment clause. Recently, the toy manufacturer decided to abandon its line of steel toys, so it made an assignment of its rights and delegation of its duties under the contract to a toymaker many times larger. The large toymaker notified the steel processor of the assignment and relayed to the processor its good faith belief that its requirements will approximate those of the assignor. Must the steel processor supply the requirements of the large toymaker? A. Yes, because there was no nonassignment clause in the contract. B. Yes, because the large toymaker acted in good faith to assure the steel processor that its requirements will approximate those of the small manufacturer into whose shoes it stepped. C. No, because requirements contracts are not assignable under the UCC D. No, because the steel processor did not give prior approval of the assignment.

B. Because the large toymaker acts in good faith in setting its requirements to approximately those of the small manufacturer into whose shoes it stepped, the contract may be assigned. The contract in this question is a "requirements" contract: The steel processor must sell the small manufacturer of children's toys all the specialized steel it requires for its toys. Generally, the right to receive goods under a requirements contract is not assignable because the obligor's duties could change significantly. In fact, here, a significant change would seem possible because the large toymaker is a larger company than the small manufacturer and its needs could be greater. However, the UCC allows the assignment of requirements contracts if the assignee acts in good faith not to alter the terms of the contract. [UCC §2-306] (The UCC applies here because goods are involved.) Thus, assuming the large toymaker's requirements remain about the same as the small manufacturer's requirements, the steel processor would be required to honor its contract, now assigned to the large toymaker. (A) is wrong because requirements contracts may be nonassignable, even without a nonassignment clause. Thus, the clause would be irrelevant. The only thing that could allow assignment of a requirements contract is a good faith limitation, as addressed in choice (B). (C) is wrong because the UCC does allow requirements contracts to be assigned, as long as the good faith limitation is satisfied. (D) is similarly incorrect. The UCC would allow assignment without approval by the obligor if there is a good faith limitation on the requirements.

A recent law school graduate was offered a job as an aide by a state legislator. The legislator told the graduate that before she could begin working, she had to take the following loyalty oath: "I swear to uphold our state and federal Constitutions; to show respect for the state and federal flags; and to oppose the overthrow of the government by violent, illegal, or unconstitutional means." The graduate told the legislator that the oath is unconstitutional and refused to take the oath. Is the graduate correct? A. Yes, as to the promise to uphold the state and federal Constitutions. B. Yes, as to the promise to respect the flag. C. Yes, as to the promise to oppose the overthrow of the government. D. No, as to all three provisions.

B. The graduate is correct as to the promise to respect the flag, but incorrect as to the other two promises. The Supreme Court has upheld oaths requiring government employees to oppose the violent overthrow of the government and to support the Constitution; hence, (A) and (C) are incorrect. However, it has held that government employees cannot be required to show respect for the flag, as a person might refuse to salute the flag on religious grounds. Thus, (B) is correct and (D) is incorrect.

A town adopted an ordinance providing that a person must have been a resident of the town for at least one year to be eligible to vote in school board elections. A resident who moved to the town seven months ago attempted to register to vote in the school board elections scheduled for the next month. However, the town clerk refused to register the resident because he will not have resided in the town for a full year prior to the election. The resident filed a class action suit on behalf of all of the new residents of the town, challenging the validity of the one-year residency requirement. Which of the following statements is correct? A. If the resident's suit is not heard before the election, it will be dismissed as moot, because the resident will have met the residency requirement by the time of the next annual election. B. The resident will prevail even if the matter is not decided until after next month's election. C. As long as there is some legitimate purpose for the one-year residency requirement, such as the need to prepare voting lists, the residency requirement will be upheld. D. The resident will lose because one-year residency requirements have been held to be permissible restrictions on the right to vote in local elections.

B. The resident will prevail even if the matter is not decided until after the election, because the suit is not moot and the residency requirement is unconstitutional. The resident's suit is not moot even if the matter will not be decided until after the election because other members of the class might have a live controversy. Under the case and controversy requirement of the Constitution, there must be a real, live controversy at all stages of the suit. If through the passage of time, the controversy between the parties is resolved, the case is said to be moot. However, there are exceptions to the mootness doctrine. In a class action, it is not necessary that the suit by the named plaintiff be viable at all stages, as long as the claim is viable by some member of the class. Thus, the suit here would not be moot. Moreover, the residency requirement here violates the resident's fundamental rights to vote and to interstate travel. A restriction on the right to vote is subject to strict scrutiny and is valid only if it is necessary to achieve a compelling state interest (otherwise the restriction violates the Equal Protection Clause by treating new residents differently from old residents). Relatively short residency requirements (e.g., 30 days) have been upheld as being necessary to promote the compelling interest of assuring that only bona fide residents vote. However, the Supreme Court has struck down longer durational requirements for lack of a compelling justification. Thus, the one-year requirement here probably unconstitutionally impinges on the right to vote. The residency requirement also impinges on the fundamental right to travel in the same manner (i.e., it discourages people from migrating by denying them the right to vote without a compelling reason). Thus, the requirement is invalid. (A) is incorrect because, as indicated above, the case will not be moot since other members of the class might have a live controversy. (C) is incorrect because it applies the wrong standard. Because fundamental rights are affected by the residency requirement here, the government must show a compelling justification; a mere rational or legitimate basis is not enough. (D) is incorrect because, as stated above, the Supreme Court has found that there was no compelling interest for a one-year residency requirement in order to vote.

The defendant was on trial for murder. The defendant called a witness to testify to an alibi. On cross-examination of the witness, the prosecutor asked, "Weren't you on the jury that acquitted the defendant of another criminal charge?" What is the best reason for sustaining an objection to this question? A. The question goes beyond the scope of direct examination. B. The probative value of the answer would be substantially outweighed by its tendency to mislead. C. The question is a leading question. D. Prior jury service in a case involving a party renders the witness incompetent.

B. This question raises several different issues: competency of witnesses, use of leading questions on cross-examination, the proper scope of cross-examination, and the probative value/prejudicial impact balancing test. Through a process of elimination, (B) emerges as the correct answer. (D) is incorrect. Under the Federal Rules, virtually all witnesses with personal knowledge are competent to testify. [Fed. R. Evid. 601] A witness is not rendered incompetent simply by having served on a jury in a prior case involving a party to the current suit. Such prior jury service might render the witness's testimony unpersuasive, but it would not make it inadmissible. (C) is incorrect because ordinarily, leading questions are permitted on cross-examination. [Fed. R. Evid. 611(c)] The prosecutor's question is a leading question, but that is perfectly permissible, especially in a case like this, where the alibi witness is not "friendly" toward the prosecution. (A) is incorrect because cross-examination is generally limited in scope to the subject matter of the direct examination and matters affecting the credibility of the witness [Fed. R. Evid. 611(b)], and the prosecutor's question is, in a roundabout way, an attempt to impeach the witness's credibility. The implication behind the question is that if the witness had served on a jury that acquitted the defendant of another criminal charge, the witness would be inclined to think the defendant innocent of the pending charge. Alternatively, the implication behind the question could be that the witness is the kind of person who is "soft on crime" and for that reason is not a credible witness. In either event, because the question is an attempt to impeach the witness's testimony, it is within the proper scope of cross-examination. This leaves (B) as the remaining correct answer. (B) is not unquestionably correct, because the probative value/prejudicial impact balancing test found in Rule 403 is weighted heavily toward admission of evidence. For evidence to be excluded under this balancing test, its probative value must be substantially outweighed by its prejudicial impact. Nevertheless, in this case, a plausible reason for sustaining an objection to the prosecutor's question is that the probative value of the answer would be substantially outweighed by its tendency to mislead. The question and answer would inevitably let the jury know that the defendant had been previously charged with a crime. This information could be highly prejudicial to his defense. Because the question and answer have little probative value (the negative inferences pertaining to the witness's credibility being very weak), it is reasonable to sustain an objection to the question on the basis that its probative value is substantially outweighed by its prejudicial impact.

A skier broke his leg when he was knocked down by the chair lift as he tried to avoid other skiers who had fallen off while disembarking. The ski resort employee operating the lift had not been paying attention and had failed to stop the lift. Ski patrol personnel placed the skier on a stretcher, which they then hooked up to a snowmobile to bring him down the mountain. The route down ran along the edge of a ski trail. Midway down, a novice snowboarder tried to see how close he could come to the stretcher without hitting it, but he lost control and landed on top of the skier's leg, damaging it further. The skier filed a lawsuit against the snowboarder and the resort in a jurisdiction that has adopted a comparative contribution system in joint and several liability cases. At trial, the skier's physician testified that the skier's leg was permanently disabled, but that neither injury, by itself, would have caused the permanent disability and it was impossible to quantify how much each injury contributed to the disability. The jury determined that the damages from the permanent disability equaled $2 million, and that the snowboarder and the resort were each 50% at fault. What amount of damages can the skier recover from the snowboarder for his permanent disability? A. $1 million, because the jurisdiction follows comparative contribution rules. B. $2 million, because it was not possible to identify the portion of the injury that the snowboarder caused. C. Nothing for his permanent disability, because the skier has not met his burden of proof as to the amount of damages that the snowboarder caused. D. Nothing for his permanent disability, because the injury inflicted by the snowboarder, by itself, would not have caused the disability.

B. $2 million, because it was not possible to identify the portion of the injury that the snowboarder caused. The skier can recover $2 million from the snowboarder because the snowboarder is jointly and severally liable for the injury. The doctrine of joint and several liability provides that when two or more tortious acts combine to proximately cause an indivisible injury to plaintiff, each tortfeasor will be jointly and severally liable for that injury. This means that plaintiff can recover the entire amount of his damages from any one defendant. The doctrine applies even though each tortfeasor acted entirely independently and at different times. Here, both the snowboarder and the employee of the ski resort breached their duty to the skier to act with reasonable care. Each tortfeasor's act was the actual cause of the skier's disability because but for either one of the acts, his leg would not have been permanently disabled. The snowboarder's act was the proximate cause of the skier's disability because the disability was the direct result of the snowboarder's act. The fact that the extent of the harm was unforeseeable is irrelevant; i.e., the tortfeasor takes the victim as he finds him. Thus, the skier can recover the entire $2 million from the snowboarder. (A) is incorrect because the contribution rules govern only whether a defendant required to pay more than his share of damages has a claim against the other jointly liable parties for the excess. Contribution does not involve the amount of damages that the plaintiff can collect in the first place. (C) is incorrect because the skier has met his burden of proof by establishing that the snowboarder was an actual and proximate cause of his permanent disability. Because the injury caused by the tortfeasors was not divisible, under joint and several liability rules, the snowboarder is liable for the full amount of the damages, including that attributable to the permanent disability. (D) is incorrect because but for the snowboarder's collision with the skier, the skier would not have been disabled. The "but for" test applies in concurrent cause situations—cases where several acts combine to cause the injury, but none of them standing alone would have been sufficient. The fact that the snowboarder's act standing alone would not have caused the disability is irrelevant to the snowboarder's liability.

The owner of a boat took two friends out on a lake near his home. One of his friends was driving the boat when it struck a partially submerged rock that the owner of the boat had forgotten to tell him about. The owner of the boat and the other passenger were injured; the driver of the boat was not hurt. In a jurisdiction that applies joint and several liability with comparative contribution, the passenger brought suit against both the boat owner and the driver, and the boat owner also sued the driver. The jury determined that the boat owner was 55% at fault and suffered $10,000 in damages, the driver of the boat was 45% at fault, and the injured passenger suffered $100,000 in damages. After entry of judgment, the boat owner paid the passenger her total damages of $100,000, while the driver of the boat has paid nothing. How much, if anything, can the boat owner recover from the driver? A. $45,000, because the driver was 45% at fault. B. $49,500, because the driver was 45% at fault and the boat owner suffered $10,000 in damages. C. $50,000, because the boat owner and the driver are jointly liable. D. Nothing, because the boat owner was more at fault than the driver.

B. $49,500, because the driver was 45% at fault and the boat owner suffered $10,000 in damages. The boat owner can recover $45,000 through comparative contribution for the passenger's claim and $4,500 on his own claim against the driver of the boat. Most comparative negligence states have adopted a comparative contribution system based on the relative fault of the various tortfeasors. Nonpaying tortfeasors who are jointly and severally liable are required to contribute only in proportion to their relative fault. Here, because the jurisdiction retained joint and several liability, the boat owner had to pay the passenger all of her damages. Under comparative contribution rules, the boat owner can obtain contribution from the driver for 45% of that amount, because the driver was 45% at fault. In addition, the boat owner has a direct claim against the driver for his own damages of $10,000, reduced by 55%, the amount of his fault. Thus, the total amount that the boat owner can recover from the driver is $49,500, making (B) correct and (A) incorrect. (C) is incorrect because it reflects traditional contribution rules, in which all tortfeasors were required to pay equal shares regardless of their respective degrees of fault. (D) is incorrect because a tortfeasor who was jointly and severally liable is not precluded from recovering contribution merely because he was more at fault than the other tortfeasors.

Several members of a small terrorist group are on trial in federal court for conspiring to bomb a military installation. The prosecution would like to introduce the testimony of a military guard at one of the installation's gates. The guard had been present when a bomb that was being planted by a member of the group had exploded prematurely. The guard will testify that she ran over to administer first aid to the member, who in great pain told her that his group was in the process of planting three other bombs in other areas of the military installation and was going to detonate them all at the same time to get publicity for their cause. The guard will also testify that the member disclosed the locations of the other bombs and the names of two other members of the group. The authorities were able to prevent the other bombings and arrest the other members of the group. The member died from his injuries. What is the best basis for allowing the guard to testify as to the member's statements? A. As a statement of a party-opponent's co-conspirator. B. As a statement against interest. C. As a statement of present state of mind. D. As a dying declaration.

B. As a statement against interest. The member's statements are admissible as a statement against interest. Under the Federal Rules, statements of a person, now unavailable as a witness, against that person's pecuniary, proprietary, or penal interest when made are admissible as an exception to the hearsay rule. Here, the member's statements implicating himself in the bombing conspiracy were against his penal interest when he made them; hence, they are probably admissible under that exception. (A) is wrong because for a co-conspirator's statement to qualify as a vicarious statement of an opposing party, the statement must have been in furtherance of the conspiracy by a participant in it. Here, the member's statements were not made in furtherance of the conspiracy but instead served to thwart its success. (C) is wrong because the member's statements are not being used to show his then-existing state of mind but rather the scope of the conspiracy and the defendants' participation in it. (D) is wrong because, even assuming that the member made the statements while believing his death was imminent (which the facts do not clearly establish), dying declarations are admissible under the Federal Rules only in a prosecution for homicide or in a civil action, and this case was neither of those.

The plaintiff is suing the defendant for personal injuries suffered when she was struck by the defendant's car. The plaintiff testified regarding the extent of her injuries. The defendant seeks to testify that when she visited the plaintiff in the hospital, the plaintiff said that if the defendant paid all of her medical bills and gave her $25,000, she would not institute legal proceedings. The plaintiff's attorney objects to the offered testimony. How should the judge rule on the admissibility of the testimony? A. Inadmissible, because it is hearsay. B. Inadmissible, because it was an offer of compromise. C. Admissible, because it is a statement against interest and, thus, an exception to the hearsay rule. D. Admissible, because it is relevant to show that the plaintiff tried to "blackmail" the defendant, and does not really have a claim.

B. Inadmissible, because it was an offer of compromise. The judge should rule the testimony inadmissible. Evidence of compromises or offers to compromise is inadmissible to prove or disprove the validity or amount of a disputed claim. (A) is incorrect; the evidence is not hearsay because it is a statement made by and offered against a party. (C) is incorrect because the statement against interest exception requires that the witness be unavailable. Here, the plaintiff is available to testify. (D) may be relevant, but the facts show that it was merely an offer to compromise a disputed claim, and therefore public policy encouraging settlements precludes the introduction of such evidence. Thus, (B) is correct.

A car collector bought a car with gold leaf paint from a manufacturer. During the first month, all the gold leaf paint peeled off. The collector sued the manufacturer, and during discovery served an interrogatory asking the manufacturer to identify all other purchasers of the gold leaf paint over the previous 10 years. The manufacturer was aware that only about 25 of the 2 million buyers of its cars have ordered the gold leaf option. The manufacturer has retained copies of all sales forms, but has not maintained separate files of the buyers of each particular option. In a court using the federal rules, what are the manufacturer's obligations with respect to the collector's interrogatory? A. It must search its files and then disclose the information. B. It may allow the collector to search the records himself. C. It may respond by stating that only about 25 of the 2 million buyers ordered the option. D. It may state that searching the records is too great a burden and so it is excused from answering the interrogatory.

B. It may allow the collector to search the records himself. Assuming the interrogatory is otherwise proper, the manufacturer may search the 2 million order forms itself or it may allow the collector access to the files. In a situation where desired information may be ascertained from the business records of the party on whom the interrogatory was served, and where the burden of finding the information is substantially the same for the party serving the interrogatory as for the party served, it is a sufficient answer to provide the serving party reasonable opportunity to examine the records. [Fed. R. Civ. P. 33] (A) is incorrect because instead of searching the files itself, the manufacturer can allow the collector access to do it. (C) is incorrect because it is not responsive to the interrogatory. (D) is incorrect because a party is not relieved of its duty to answer just because the search is burdensome.

A patient sued a surgeon for medical malpractice, alleging that the surgeon used an improper procedure during cardiac surgery, thereby permanently injuring the patient. The surgeon then wrote to a colleague, who was a renowned cardiologist, asking whether in her opinion the procedure was proper. In response, the colleague stated that the procedure was likely improper, but that she was unsure whether the patient's injuries resulted from it. After receiving the colleague's letter, the surgeon did not have any further communication with her about the matter. Is the letter discoverable? A. No, because the colleague did not have first-hand knowledge regarding the surgery that she was evaluating. B. No, because the colleague will not testify at trial on behalf of the doctor. C. Yes, because the colleague was not retained or specially employed by the doctor. D. Yes, because the colleague is not an expert.

B. No, because the colleague will not testify at trial on behalf of the doctor. The letter is not discoverable because the colleague is a nontestifying expert. An expert is a person with knowledge, skill, experience, training, or education in a particular field. The opinion of an expert consulted in anticipation of litigation is discoverable if a party intends to call the expert as a trial witness. Conversely, the opinion of an expert who is retained in anticipation of litigation but who is not expected to testify at trial (i.e., a consulting expert) is discoverable only upon a showing of exceptional circumstances under which it is impracticable to obtain facts or opinions by other means. Absent such circumstances, these opinions are not discoverable, regardless of whether the nontestifying expert was informally consulted or whether she was retained or specially employed. Thus, the doctor need not produce the letter from his colleague, an expert in cardiology, because the doctor does not expect to call her as a trial witness. (A) is incorrect because an expert need not have first-hand knowledge of facts to give an opinion based on those facts. In any event, although a fact witness's lack of knowledge may in some circumstances provide a reason for excluding testimony at trial, it is not a valid ground for resisting discovery of relevant information. (C) is incorrect because the opinion of a nontestifying expert is nondiscoverable regardless of whether she is formally retained or not. (D) is incorrect because the cardiologist is an expert. The question states that she is a renowned cardiologist and that the doctor is consulting her about a cardiac surgery.

A statute requires that any pilot who flies passengers for hire must have a commercial pilot's license. An experienced pilot who had only a private pilot's license and not the commercial license required by statute was hired by an attorney to fly her to another city to close a deal. The attorney knew that the pilot did not have a commercial license but the only commercial flight to the city was at an inconvenient time. The pilot flew the attorney through bad weather and landed safely, but because of a minor navigational error he landed at an airport a few miles away from the airport he was heading for. As he was going to start taxiing toward the hangar, another plane struck the aircraft. The student pilot of that plane had ignored the control tower's instructions and gone onto the landing runway instead of the takeoff runway. The attorney was injured in the collision. If the attorney sues the pilot for her injuries, who will prevail? A. The pilot, because the attorney knew he lacked a commercial license and voluntarily assumed the risk of flying with him. B. The pilot, because the injuries to the attorney were caused by the negligence of the student pilot of the other plane. C. The attorney, because the pilot violated a statute designed to prevent persons without commercial licenses from flying passengers for a fee, and such violation imposes liability per se. D. The attorney, because the pilot landed at the wrong airport, and but for this mistake the attorney could not have been injured by the other aircraft.

B. The pilot, because the injuries to the attorney were caused by the negligence of the student pilot of the other plane. The pilot will prevail because the conduct of the other plane's student pilot constituted a superseding intervening force that relieves the pilot from liability. To establish a prima facie case for negligence, the attorney must show that the pilot's breach of his duty to her was the actual and proximate cause of her injury. The attorney can establish actual cause because but for the pilot's error, she would not have been injured. However, not all injuries "actually" caused by a defendant will be deemed to have been proximately caused by his acts. The general rule of proximate cause is that the defendant is liable for all harmful results that are the normal incidents of and within the increased risk caused by his acts. This rule applies to cases such as this, where an intervening force comes into motion after the defendant's negligent act and combines with it to cause plaintiff's injury (indirect cause cases). Here, the pilot's navigational error did create a greater risk of collision with other planes in the process of landing, but it did not increase the risk of a plane using the landing runway to take off in disregard of the control tower's instructions once the pilot was safely on the ground. Hence, the student pilot's unforeseeable conduct was not within the increased risk created by the pilot's negligence and constitutes a superseding force that breaks the causal connection between the pilot's conduct and the attorney's injury, enabling the pilot to avoid liability to the attorney. (A) is incorrect because assumption of the risk requires knowledge of the specific risk and the voluntary assumption of that risk. Although the attorney knew that the pilot lacked a commercial license, she also was under the impression that he was a very good pilot. There is no indication that she knew of or voluntarily assumed any risk. Certainly, she did not assume the risk of the type of harm she suffered. (C) is incorrect because even though the pilot may be negligent per se, he would not be liable per se. A clearly stated specific duty imposed by a statute may replace the more general common law duty of due care when: (i) the plaintiff is within the class to be protected by the statute, and (ii) the statute was designed to prevent the type of harm suffered. The statutory duty is clearly stated and arguably applies here because the attorney, as the pilot's paying passenger, is within the protected class, and runway collisions and other pilot errors are what the license requirement is officially designed to prevent. There are no grounds for excusing the pilot's violation of the statute, so the pilot's conduct could be seen as "negligence per se." This means that plaintiff will have established a conclusive presumption of duty and breach of duty. However, for the attorney to prevail, she must also establish actual and proximate causation. As explained above, the attorney will not be able to show that the pilot's negligence was the proximate cause of her injuries. Thus, while she may be able to establish "negligence per se," she has not made a case for "liability per se." (D) is true as far as it goes. An act or omission to act is the cause in fact of an injury when the injury would not have occurred but for the act, and this injury would not have occurred but for the pilot's landing at the wrong airport. However, the attorney must also be able to establish that the pilot's conduct was a proximate cause of her injury. As noted above, the pilot's conduct was not a proximate cause of her injury because the student pilot's actions acted as a superseding intervening force.

After a sporting event at a stadium, one of the fans sought out the referees to complain about their handling of the game. The fan took out an electronically amplified bullhorn and knocked on the door of the referees' room. When one of the referees opened it, the fan began yelling and berating the referee through the bullhorn. The referee slammed the door shut, striking the bullhorn and jamming it against the fan's mouth, knocking out two of his teeth. If the fan asserts a claim based on battery against the referee and the referee prevails, what is the likely reason? A. The referee did not foresee that the bullhorn would knock out the fan's teeth. B. The referee did not know that the door was substantially certain to strike the bullhorn. C. The referee was entitled to use force to protect himself. D. The fan's conduct provoked the referee's response.

B. The referee did not know that the door was substantially certain to strike the bullhorn. If the referee prevails, it will be because he did not know the door would strike the bullhorn, so he did not have the intent to commit a battery. Battery requires: (i) an act by defendant that causes a harmful or offensive contact to plaintiff's person; (ii) intent to cause the harmful or offensive contact; and (iii) causation. Here, there was a harmful contact caused by the referee. The only consideration is whether the referee had the requisite intent. If a person knows with substantial certainty the consequences of his action, he has the intent necessary for this type of tort. If the referee did not know that the door was substantially certain to hit the bullhorn the fan was holding, the referee did not have the intent necessary for battery. (For purposes of battery, anything connected to or being held by the plaintiff is usually considered part of the plaintiff's person.) (A) is wrong because if the referee intended to cause a harmful contact (a battery), he is liable for all of the consequences of his actions, whether he intended them or not. A defendant need not foresee the extent of the injuries caused by his intentional act to be held liable for them. (C) is wrong because this is not a case of self-defense. Self-defense is appropriate when a person reasonably believes that he is being or is about to be attacked. Nothing in the facts shows any basis for the referee to believe that the fan was going to harm him. Thus, self-defense is not appropriate here. (D) is wrong because it does not provide the referee with a defense. The fan's conduct angered the referee and may have triggered his actions, but because the fan's conduct was not sufficient to allow the referee to act in self-defense, the referee's use of force here is not excused.

A seller conveyed her residential city property to a buyer by a general warranty deed. On taking possession of the property, the buyer discovered that the garage of his neighbor encroached six inches onto his property. If the buyer wishes to compel the seller to assist him in a suit against the neighbor, which of the following covenants may he rely on to do so? A. Seisin and encumbrances. B. Warranty and further assurances. C. Seisin and warranty. D. Encumbrances and further assurances.

B. Warranty and Further Assurances The buyer would rely on the covenants of warranty and further assurances to compel the seller to assist him in a suit against his encroaching neighbor. Under the covenant of warranty, the grantor agrees to defend, on behalf of the grantee, any lawful or reasonable claims of title by a third party, and to compensate the grantee for any loss sustained by the claim of superior title. The covenant for further assurances is a covenant to perform whatever acts are reasonably necessary to perfect the title conveyed if it turns out to be imperfect. These covenants are "continuous" (run with the land) and require the grantor to assist the grantee in establishing title. The covenants of seisin and encumbrances do not require such assistance. A covenant of seisin is a covenant that the grantor has the estate or interest that she purports to convey. Both title and possession at the time of the grant are necessary to satisfy this covenant. The covenant against encumbrances is a covenant assuring that there are neither visible encumbrances (easements, profits, etc.) nor invisible encumbrances (mortgages, etc.) against the title or interest conveyed. While the seller may have violated these two covenants because of the garage encroachment, they do not provide the basis to compel her to assist the buyer in a title suit. Instead, the buyer merely has a cause of action against the seller for their breach. Therefore, (A), (C), and (D) are wrong.

A defendant was charged with arson (a felony) of an antique shop. Only one corner of the shop was damaged before the fire was extinguished. Under a plea agreement, the defendant pled guilty and received a suspended sentence. Because the owner of the shop had not yet insured a recently acquired 400-year-old refectory table that was destroyed by the fire, he sued the defendant for damages. At trial, the owner offers the properly authenticated record of the defendant's conviction for arson. Should the record be admitted into evidence? A. Yes, as proof of the defendant's character in order to infer liability. B. Yes, as proof that the defendant set the fire. C. No, because the conviction was not the result of a trial. D. No, because it is hearsay not within any exception.

B. Yes, as proof that the defendant set the fire. The record of the defendant's conviction should be admitted to prove that the defendant set the fire. The record of the conviction is hearsay; i.e., it is a statement, other than one made by the declarant while testifying at the trial or hearing, offered to prove the truth of the matter asserted. Under the Federal Rules, however, such judgments fall within the hearsay exception for records of felony convictions. Under the Federal Rules, judgments of felony convictions are admissible in both criminal and civil actions to prove any fact essential to the judgment, whether the judgment arose after trial or upon a plea of guilty. [Fed. R. Evid. 803(22)] For purposes of this Rule, a felony is any crime punishable by death or imprisonment in excess of one year. Arson is a felony. Consequently, a properly authenticated copy of the defendant's conviction of this crime is admissible to prove the fact that the fire that destroyed the table was set by the defendant, a fact essential to the judgment of conviction. Note that the actual plea of guilty is also admissible as a statement of a party-opponent (commonly called an admission). This type of judicial statement is not conclusive, and the defendant may explain the circumstances of the plea. The plea, being an opposing party's statement, is nonhearsay under the Federal Rules. (A) is incorrect because, in a civil case, evidence of character to prove the conduct of a person in the litigated event is generally not admissible. Circumstantial use of prior behavior patterns for the purpose of inferring that, at the time and place in question, a person probably acted in accord with such patterns creates a danger of prejudice and distraction from the main issues. Therefore, the record of the conviction cannot be used to infer liability by showing the defendant's character. (C) is incorrect because, as noted above, a judgment of a felony conviction is admissible under Federal Rule 803(22) regardless of whether the conviction resulted from a trial or a guilty plea. (D) is incorrect because, as discussed above, the judgment is within the exception to the hearsay rule for records of felony convictions.

A man and a woman were arrested and charged with a series of armed robberies. Each suspect was given Miranda warnings, and different interrogation teams questioned each suspect separately. Upon being questioned, the man told the police, "I'm not going to talk until I see a lawyer." An officer responded, "You might want to reconsider, because your partner has already confessed, and she's implicated you in the crimes." The man then told the police that he wanted to talk to the woman privately. The police escorted the man to the woman's cell, locked him in with her, and left. Unbeknownst to either of them, the police had bugged the woman's cell and recorded both the man and the woman making self-incriminating statements during their meeting. The man made no further statements to the police on advice of counsel, whom he called immediately after his conversation with the woman. The man was put on trial first, and the prosecution sought to introduce into evidence tapes of the bugged conversation between the man and the woman. The defense made a motion to suppress the evidence. Should the court grant the motion to suppress? A. Yes, because the evidence is the fruit of a wiretap that violated the Fourth Amendment. B. Yes, because the police created a situation likely to induce the defendant to make an incriminating statement. C. No, because there is no expectation of privacy in a jail cell. D. No, because the conversation constituted a waiver of the man's Miranda rights.

B. Yes, because the police created a situation likely to induce the defendant to make an incriminating statement. The conversation should be suppressed because the police conduct violated the man's Sixth Amendment right to counsel. The Sixth Amendment provides that in all criminal prosecutions a defendant has a right to the assistance of counsel at all critical stages after formal proceedings have begun. For Sixth Amendment purposes, a criminal prosecution begins when adversary judicial proceedings have commenced, such as the filing of formal charges in this case. Because custodial interrogation is a critical stage of prosecution, the Sixth Amendment is violated by post-charge interrogation unless the defendant has waived his right to counsel. Interrogation includes not only direct questioning, but also any other conduct by the police intended to elicit a response. The police conduct here (telling the man that the woman had implicated him and then bugging the conversation) constitutes prohibited interrogation. [See Maine v. Moulton (1985)] (A) is incorrect because the wiretap was not an illegal search under the Fourth Amendment. Wiretapping and other forms of electronic surveillance are subject to the Fourth Amendment prohibition of unreasonable searches and seizures. However, to have a Fourth Amendment right, a person must have a reasonable expectation of privacy with respect to the place searched or the item seized. In a different context, the Supreme Court has held that prisoners have no reasonable expectation of privacy in their cells or in any personal property that they have in their cells. [Hudson v. Palmer (1984)] Hence, neither defendant can assert a Fourth Amendment claim based on the wiretap, because they had no reasonable expectation of privacy in the jail cell. The fact that there was no expectation of privacy does not make choice (C) correct, however. Even though he probably cannot claim that the bugging was an unreasonable search under the Fourth Amendment, the man can claim that it was an interrogation in violation of his Sixth Amendment right to counsel, as discussed above. (D) is incorrect because it is irrelevant. The facts probably would not give rise to a Miranda violation in light of the Court's ruling in Illinois v. Perkins (1990) that Miranda does not apply unless interrogation is by someone known to be a police officer (on the rationale that Miranda is merely a prophylactic rule designed to offset the coercive nature of a custodial interrogation by a police officer). In any case, Miranda rights and Sixth Amendment rights to counsel can only be waived knowingly, and so the man's ignorance of the fact that the cell was bugged precludes a finding of waiver here.

A backgammon player was upset after losing a match against the club champion. Rushing out of the club, he inadvertently grabbed the champion's board, which looked very much like his own but which was much more expensive. The player left the backgammon board in the trunk of his car, as was his usual practice. During the night, the car was stolen and along with it, the champion's expensive backgammon board. In an action by the champion against the player to recover the value of the backgammon board, is the champion likely to recover? A. Yes, because when the player took the backgammon board he committed a trespass to the champion's chattel. B. Yes, because when the backgammon board was stolen along with the car, the player became liable for conversion of the champion's chattel. C. No, because the player believed in good faith that the board was his when he took it from the backgammon club. D. No, because the backgammon board was lost through no fault of the player's.

B. Yes, because when the backgammon board was stolen along with the car, the player became liable for conversion of the champion's chattel. The champion will recover from the player for conversion. Conversion consists of (i) an act by defendant interfering with plaintiff's right of possession in the chattel, (ii) intent to perform the act bringing about the interference with plaintiff's right of possession, (iii) causation, and (iv) damages—an interference that is serious enough in nature or consequence to warrant that the defendant pay the full value of the chattel. Intent to trespass is not required; intent to do the act of interference with the chattel is sufficient for liability. Therefore, the player was guilty of conversion when he intentionally (i.e., volitionally) took the champion's board, which resulted in its loss, even though the player did not intend to lose it or even realize that he had taken the property of another. (A) is not the best answer because complete loss of a chattel, permitting the plaintiff to recover its full value, is too serious an interference to be classified a mere trespass. Trespass to chattels consists of: (i) an act by defendant interfering with plaintiff's right of possession in the chattel, (ii) intent to perform the act bringing about the interference with the plaintiff's right of possession, (iii) causation, and (iv) damages. Had the champion been able to recover the board, and had he been able to show actual damages during the time of dispossession, he might have been able to recover for trespass to chattels. (C) is wrong because the player's good faith is irrelevant. Even if the conduct is wholly innocent, liability will attach when the interference with the chattel is serious in nature. (D) is wrong because the fact that the player's car was stolen does not relieve him of liability. His initial trespassory interference with the champion's backgammon set was a substantial factor in its complete loss, because it would not have otherwise been in the trunk of his car. Thus, the causation element for conversion is satisfied.

A 15-year-old sophomore high school student became pregnant, and the school board required her to attend a special program for pregnant students instead of her regular classes. The girl did not want to attend a special program; rather, she wanted to attend her regular classes.She sued the school district in federal district court, demanding that she be allowed to attend her regular classes. Before her case came to trial, the girl gave birth to the child. Subsequently, the district reinstated her in her regular classes.When her suit comes before the federal district court, what should the court do? A)Dismiss the action, because she is no longer pregnant. B)Dismiss the action, because she is no longer required to attend the special classes. C)Hear the case on the merits, because she may get pregnant again before she graduates from the high school. D)Hear the case on the merits, because it impacts the right to privacy, which is an important federal issue.

C.

The federal government recently constructed a radio telescope in Puerto Rico. In addition to receiving radio signals, the telescope was set up to beam radio waves far out into space. As part of the bill providing for operational funding for the facility, Congress provided for a program to "inform any aliens who might be listening in outer space of the 'American Way of Religion.'" A $10 million appropriation was provided; any religious group whose membership exceeded 500 members in the United States was permitted to prepare a five-minute presentation, and the federal government would pay for the recording of the presentations and broadcast them into space using the transmitter in Puerto Rico. The President signed the bill and it became law. A religious group with a large following in Europe, but only 100 members in the United States, protested and filed suit. Will the court find the religious broadcasts to be constitutional? A. Yes, because the power to regulate commerce with foreign nations and among the several states implies that commerce with outer space is included as well. B. Yes, because the commerce power is not limited by First Amendment prohibitions. C. No, because the requirement of a minimum number of members violates the First Amendment. D. No, because the broadcasts are a waste of money and exceed the spending powers of Congress.

C. The court will find that the requirement of a minimum number of members violates the Establishment Clause of the First Amendment. The Establishment Clause prohibits any law "respecting an establishment of religion." While usually a three-part test based on Lemon v. Kurtzman is used to determine whether legislation creates improper government involvement with religion, the "compelling government interest" test is used if a law or government program discriminates among religions. Here, the law differentiates among different religious groups, allowing only those with larger memberships to record presentations. There is no compelling government interest for discriminating among the religious groups in this way; thus the legislation is unconstitutional. (A) is incorrect despite the fact that the federal power to regulate commerce could be interpreted as extending to commerce with outer space should the occasion arise. Nevertheless, the commerce power does not override independent constitutional restrictions (e.g., the Establishment Clause) on the conduct in question here. (B) is an incorrect statement of law. The federal commerce power cannot be used to abrogate freedom of speech or to discriminate in favor of religious groups. An exercise of the commerce power generally is subject to all limitations placed on government by the Constitution, including those of the First Amendment. (D) is incorrect. Regardless of merit, almost all expenditures made by Congress are permissible under its spending power. [U.S. Const. art. I, §8] Rather than limit the power only to spending for accomplishment of other enumerated powers, this provision grants Congress broad power to spend for the "general welfare" (i.e., any public purpose). As long as the expenditure is not conditioned on requiring a recipient to forgo an individual constitutional right, it is within the spending power of Congress.

A department store buyer and a manufacturer of food processors entered into a written contract whereby the manufacturer would sell to the buyer 50 of its top-of-the-line models for $100 each. When the delivery arrived on May 15, several days early, the buyer noticed that the food processors were a different model that did not have all of the features as the top-of-the-line model that was ordered. The buyer contacted the manufacturer and told him that he was rejecting the food processors that were delivered to him and expected the manufacturer to send 50 top-of-the-line models immediately. The manufacturer replied that because of a backlog of orders that had not yet been filled, the top-of-the-line models could not be delivered until August 15. Because the department store had contracted with a restaurant to deliver three top-of-the-line models by May 31, the buyer delivered three of the nonconforming food processors along with a promise to replace them with three top-of-the-line models in mid-August. The buyer returned the remaining food processors to the manufacturer. How much could the department store recover from the manufacturer for the three food processors that it delivered to the restaurant? A. Nothing, because they were resold to another. B. Nothing, because it accepted them knowing they were defective. C. The difference between the market price of the top-of-the-line models and the existing food processors' actual value. D. The difference between the existing food processors' actual value and the cost of the food processors that the department store must provide to the restaurant in mid-August.

C. The department store was entitled to recover contract damages from the manufacturer for the three food processors that it accepted. If the buyer accepts goods that breach one of the seller's warranties, the basic measure of damages is the difference between the value of the goods as delivered and the value they would have had if they had been according to the contract, which is best stated by choice (C). The department store's acceptance of the three food processors did not waive its right to collect damages for the defect in quality. Thus, (A) and (B) are wrong. Having accepted the nonconforming food processors, the department store's damages would be the difference between the value of the food processors as received and what they would have been worth if they had been as warranted, plus foreseeable incidental and consequential damages. (D) is wrong because the measure of damages is based on market value rather than cost. Also, the agreement with the restaurant was to accommodate the department store only and was not foreseeable by the manufacturer.

A witness is called in a contract action between a plaintiff and a defendant. The witness takes his oath and testifies. During cross-examination, the defendant's attorney asked the witness this question: "Isn't it true that even though you took an oath to tell the truth so help you God, you are an atheist and don't even believe in God?" Upon the proper objection, will the judge require that the witness answer this question? A. Yes, because the question is relevant to the witness's character for truthfulness. B. Yes, because instead of taking the oath, the witness could have requested to testify by affirmation without any reference to God. C. No, because evidence of the beliefs or opinions of a witness on matters of religion is not admissible to impair credibility. D. No, because an attack on the competency of a witness must be made at the time the witness is sworn.

C. The judge should not require that the witness answer the question because evidence of the religious beliefs of a witness is not admissible to challenge credibility. Lack of religious belief is no longer a basis for excluding a witness. Not only are a person's religious convictions irrelevant in determining the competence of a witness, Federal Rule 610 provides that a witness's religious beliefs or opinions are not admissible to show that the witness's credibility is thereby impaired or enhanced. Thus, (C) is correct and (A) is incorrect. (B) is incorrect. While it is true that the witness could have requested a different type of oath, Rule 610 prohibits this type of question because it would have shown his lack of religious beliefs. (D) is incorrect because, as discussed above, lack of religious belief is no longer a basis for disqualification; thus, this would not constitute an attack on the witness's competency.

While working on a construction project, a plaintiff was injured when a heavy object struck his knee. Although the plaintiff was fully compensated for his injuries at the time of the incident, he now seeks disability payments from the construction company because he has developed arthritis in the same knee. The construction company claims that the arthritis has nothing to do with the plaintiff's on-the-job injury and refuses to pay him disability money. The plaintiff sues. A doctor takes the stand to testify for the plaintiff. He is qualified as an expert witness and during direct examination states that in his opinion the blow to the plaintiff's knee caused his arthritis. On cross-examination, the construction company's attorney produces a treatise on arthritis and asks the doctor if the treatise is considered to be authoritative. The doctor responds that the treatise is a standard authority in the field, but that he did not rely on it in forming his professional opinion regarding the plaintiff's condition. The attorney then seeks to introduce into evidence a statement in the treatise that "the idea that arthritis can be caused by a single traumatic event is purely folklore, although it is widely believed by the ignorant who have no scientific basis for their beliefs." The plaintiff's attorney objects. How should the court rule on the admissibility of the statement from the treatise? A. Admissible, but only for the purpose of impeaching the doctor's testimony. B. Admissible, but only as substantive evidence. C. Admissible, both as substantive evidence and for purposes of impeaching the doctor. D. Inadmissible.

C. Admissible, both as substantive evidence and for purposes of impeaching the doctor.

The defendant robbed a bank and fled in a getaway car driven by an accomplice, not realizing that one of the bundles of money he took had the serial numbers recorded and had a tiny tracking device attached to the wrapper. The bank's security consultant obtained portable tracking equipment and was able to trace the bundle of money to the defendant's house. The police were notified and they arrived at the defendant's house a few hours after the robbery. They knocked on the door, announced their presence, and saw someone matching the description of the robber in the hallway. They entered and arrested the defendant, and then conducted a protective sweep of the house for the accomplice, who they believed had a gun. They did not find him, but while checking a closet, they discovered several of the bundles of money from the bank and a gun the defendant had used in the robbery. The police also discovered two clear plastic bags of what appeared to be marijuana sitting on top of a dresser. They seized the money, the gun, and the two bags. Later testing confirmed that the substance in the bags was marijuana. The defendant was charged with the bank robbery and with possession of the marijuana. At a preliminary hearing, he moves to suppress introduction of the money, gun, and marijuana. How should the court rule? A. Grant the motion as to the marijuana but not as to the money or the gun because the money and gun were found as a result of the protective sweep for the defendant's accomplice. B. Grant the motion as to the money and the gun but not as to the marijuana because the bags containing the marijuana were clearly visible on the dresser during the search. C. Grant the motion as to all of the evidence seized. D. Deny the motion as to all of the evidence seized.

C. Grant the motion as to all of the evidence seized The court should suppress all of the evidence because it was the fruit of an unconstitutional arrest. As a general rule, the police must have an arrest warrant to effect an arrest of an individual in his own home. There is no general "emergency" exception to the warrant requirement. While police officers in hot pursuit of a fleeing felon or trying to prevent the destruction of evidence may sometimes make a warrantless search and seizure, the burden is on the government to show that one of those exceptions applies. Here, the police did not arrive at the defendant's house in hot pursuit of the defendant, and there was no indication that the defendant might be destroying the money or other evidence; i.e., there were no circumstances precluding them from keeping the house under surveillance while they obtained a warrant. Hence, the arrest was unconstitutional. Because an arrest constitutes a seizure under the Fourth Amendment, the exclusionary rule applies, and evidence that is the fruit of the unconstitutional arrest may not be used against the defendant at trial. Here, all of the evidence was seized without a warrant, and none of the other exceptions to the warrant requirement are applicable. While the protective sweep that turned up the money and gun probably would have been within the bounds of a search incident to an arrest because the police had reason to believe an armed accomplice was present, the arrest in violation of the Fourth Amendment makes the search unlawful. Similarly, while the bags of marijuana were discovered in plain view, the police have to be legitimately on the premises for that exception to apply. Thus, (C) is correct; (A), (B), and (D) are incorrect.

A retailer entered into an oral contract with an office supply wholesaler to buy 100 file boxes for an upcoming back to school sale at the retailer's store. The wholesaler agreed to deliver the file boxes in two weeks at a cost of $4 per file box. A week later, the retailer phoned the wholesaler and asked if she could increase her order to 200 file boxes. The wholesaler agreed. The wholesaler delivered the 200 file boxes as promised, but the retailer accepted only 150 upon discovering that she lacked storage space for all 200. May the wholesaler recover damages with respect to the 50 file boxes that were not accepted? A. Yes, because the retailer accepted $600 worth of file boxes. B. Yes, because the modification was for less than $500. C. No, because the contract as modified was for $800. D. No, because the wholesaler is a merchant with respect to file boxes.

C. No, because the contract as modified was for $800. The wholesaler may not recover damages. Under the UCC Statute of Frauds, a contract for the sale of goods for $500 or more is unenforceable unless evidenced by a writing signed by the party sought to be held liable. The original contract was for $400 and, thus, was not within the Statute. Whether a modification must be in writing to be enforceable depends on whether the entire contract price as modified is within the Statute. Here, the retailer and the wholesaler modified their original contract to 200 file boxes, bringing the total price to $800. Thus, the modification was unenforceable under the Statute of Frauds, and the wholesaler cannot collect damages with respect to the 50 unaccepted file boxes. (A) is incorrect. Acceptance is an exception to the Statute of Frauds—but only to the extent of the goods accepted. That is, an oral contract for the sale of goods for $500 or more is enforceable to the extent the goods are accepted. Here, the original contract for 100 boxes is enforceable, but the modification is not. However, since the retailer accepted 50 additional boxes, the modified contract is enforceable to the extent of the additional 50 boxes accepted. The fact that the accepted amount meets the Statute of Frauds $500 threshold does not make the contract enforceable for all 200. (B) is incorrect because, as noted above, when determining whether a contract for the sale of goods is enforceable, we look at the whole contract price as modified; the price of the modification itself does not matter. (D) is incorrect because the fact that the wholesaler is a merchant with respect to the goods being sold (file boxes) has no bearing on the enforceability of the contract here.

An attorney came to work on a Saturday. When he signed in, he was advised by the morning security guard employed by the building management that he must be out of the building by 5 p.m., when it closes. However, he stayed past 5 p.m. to complete a brief that had to be filed on Monday morning. At 5:15 p.m., the afternoon security guard set the locks on all the doors of the building and left. Because she was in a hurry, she did not check the sign-in sheet to make sure that everyone had signed out, contrary to mandatory procedures. When the attorney tried to exit 15 minutes later, he discovered that the doors were all locked and could not be opened from the inside. He used his cell phone to call for help, and a supervisor from the building arrived and let him out shortly thereafter. If the attorney sues the building management for false imprisonment, is he likely to win? A. Yes, because the guard acted recklessly by locking the doors and leaving without checking that everyone was out of the building. B. No, because the attorney became a trespasser by staying in the building past 5 p.m. C. No, because the guard did not know that the attorney was locked in the building. D. No, because the attorney suffered no harm from the confinement.

C. No, because the guard did not know that the attorney was locked in the building. The attorney will lose because the guard did not know that he was still in the building. For false imprisonment, the plaintiff must show (i) an act or omission on the part of the defendant that confines or restrains the plaintiff to a bounded area, (ii) intent on the part of the defendant to confine or restrain the plaintiff, and (iii) causation. Here, because the guard apparently did not know that the attorney was still in the building, she had no intent to confine him when she locked the doors. (A) is incorrect because recklessness is not enough; while the attorney likely has a cause of action for negligence against the guard, and through respondeat superior, the building, his claim is for false imprisonment. For liability for false imprisonment, there must be an intent to confine. (B) is incorrect because his status as a trespasser, while it may otherwise make him liable to the building for trespass, does not preclude him from recovering for false imprisonment. (D) is incorrect because the attorney need not show harm from the confinement to recover for false imprisonment, as long as he was aware of the confinement.

public high school's drug policy strictly prohibited the use, possession, or sale of any drug on school grounds, including any prescription or over-the-counter medication, unless supervised by a nurse. During lunch, the school principal observed a student ingesting two white pills. The student admitted to the principal that the pills were aspirins and had been given to her by a senior. School officials approached the senior and demanded to search her backpack. When no aspirins were found in the backpack, the officials required the senior to submit to a private physical search by the female school nurse. Some aspirins were subsequently found in the waistband of the senior's gym shorts that she was wearing under her school uniform, and she was suspended. The senior's mother sued school officials, claiming that the physical search violated her daughter's Fourth Amendment rights against unreasonable searches and seizures. In response, the school officials filed a motion for summary judgment against the mother's claim. The facts above are stipulated to by the parties. Should the court grant the motion for summary judgment? A. Yes, because the search revealed that the senior had violated the drug policy. B. Yes, because the school officials had reasonable grounds to believe that the search was necessary. C. No, because the trier of fact could determine that the search was excessively intrusive in light of the nature of the infraction. D. No, because the trier of fact could determine that the school officials did not have probable cause to conduct a physical search based on the uncorroborated statement of a minor.

C. No, because the trier of fact could determine that the search was excessively intrusive in light of the nature of the infraction. The motion should be denied. A school search will be upheld only if it offers a moderate chance of finding evidence of wrongdoing, the measures adopted to carry out the search are reasonably related to the objectives of the search, and the search is "not excessively intrusive in light of the age and sex of the student and the nature of the infraction." [Safford United School District #1 v. Redding (2009)] In the Redding case, school officials were also attempting to locate lawful pain killers believed to be on a student's person based on an uncorroborated tip from another student. The medication was prohibited on school premises without a doctor's note on file. The Court concluded that a search of the student's outer clothing and backpack did not violate the Fourth Amendment. However, a subsequent strip search of the student was unconstitutional. Here, the trier of fact could find that a search of the senior's gym shorts that she was wearing under her school uniform was excessively intrusive and not reasonably related to the objectives of the search; hence, the school officials should not be entitled to summary judgment. (A) is incorrect. The fact that a search reveals the contraband that prompted the search does not make the search permissible under the Fourth Amendment. (B) is incorrect because, not only must school officials have reasonable grounds for the search, the search must not be excessively intrusive. Because the Court has held that a strip search to locate over-the-counter pain killers was excessively intrusive, summary judgment for the school officials is inappropriate here. (D) is incorrect because the Court has observed that the school setting requires some modification of the level of suspicion of illicit activity needed to justify a search, and has ruled that only reasonable suspicion and not probable cause is required.

A woman was struck by a brick with her name scrawled on it that was thrown through her bedroom window. The victim believes that her ex-boyfriend, who is a gang member, threw the brick because she has become active in anti-gang groups, but she did not actually see him throw it. If the ex-boyfriend is arrested and put on trial for battery, which of the following items of the victim's proposed testimony is LEAST likely to be admitted? A. The victim recently moved to a new apartment and only her ex-boyfriend and a few family members knew its location. B. The victim had testified against a member of her ex-boyfriend's gang last month in a drug case. C. On another occasion, the victim had seen her ex-boyfriend throw a rock through the window of a rival street gang member. D. Immediately after the brick went through her window, the victim heard a voice she recognized as her ex-boyfriend's yell, "If you don't start minding your own business, you'll get a lot worse than this next time!"

C. On another occasion, the victim had seen her ex-boyfriend throw a rock through the window of a rival street gang member. Evidence of the defendant's other crimes or misconduct is admissible only if relevant to some issue other than the defendant's character or propensity to commit the crime charged. Such acts would be admissible to show motive, intent, absence of mistake, identity, or a common plan or scheme. Of these, the only one possibly relevant to these facts is identity. Evidence that the accused committed prior criminal acts that are so distinctive as to operate as a "signature" may be introduced to prove that the accused committed the act in question. Merely throwing an object, such as a brick, through a window could not be considered so distinctive as to operate as a signature. Thus, this evidence would not show identity. The only possible reason for offering the evidence is to show the ex-boyfriend's propensity to commit the crime charged, in which case the testimony will be inadmissible. (A) is wrong because it is circumstantial evidence that the ex-boyfriend threw the brick. It is relevant because it tends to make it more probable that he threw the brick than it would be without the evidence. (B) is wrong because it is relevant and goes to motive. It too makes it more probable that the ex-boyfriend threw the brick than it would be if the victim had not testified against a member of his gang. (D) is wrong because the victim's identification of the ex-boyfriend's voice places him at the scene and is thus relevant. It is more probable that he threw the brick than it would be in the absence of this testimony. The identification of a voice is properly authenticated by the opinion of a person familiar with the alleged speaker's voice. As his ex-girlfriend, the victim would be sufficiently familiar with the ex-boyfriend's voice to make a proper identification.

A woman was arrested, given Miranda warnings, and questioned about an armed robbery. After she asked to speak with an attorney, the police stopped questioning her about the robbery. Several hours later, the police gave the woman a fresh set of Miranda warnings and began to question her about a different robbery. She did not repeat her request for an attorney and instead made several incriminating statements about the robbery. At the woman's trial for the robbery for which she made incriminating statements, the prosecution seeks to have her statements introduced into evidence. If the woman's attorney objects on appropriate grounds, how should the court rule? A. Overrule the objection, because the police did not badger the woman into confessing. B. Overrule the objection, because the woman did not renew her request for an attorney after receiving fresh Miranda warnings. C. Sustain the objection, because the police did not honor the woman's request. D. Sustain the objection, because a confession obtained in violation of a defendant's Miranda rights but otherwise voluntary may be used against the defendant.

C. Sustain the objection, because the police did not honor the woman's request. The court should sustain the objection because the police did not honor the woman's request for an attorney. At any time prior to or during a custodial interrogation, the accused may invoke a Miranda (Fifth Amendment) right to counsel. If the accused invokes this right, all questioning must cease until the accused is provided with an attorney or initiates further questioning himself. Thus, the police questioning of the woman about the robbery was improper, and she can have her statements excluded. (A) is incorrect. After receiving Miranda warnings, if an accused invokes the right to remain silent, the police cannot badger the accused. However, courts have ruled that if the police scrupulously honor the request, they can rewarn the accused and later resume questioning, at least about a different crime. Here, however, the accused did not simply invoke the right to remain silent, but rather requested an attorney. After such a request, as indicated above, all questioning must cease. (B) is incorrect because the accused does not need to reassert the right to an attorney; all questioning must stop until the accused is provided an attorney or resumes the questioning herself. (D) is incorrect. It is stating the rule for impeachment-a confession obtained in violation of a defendant's Miranda rights but otherwise voluntary may be used against the defendant for purposes of impeachment, but there is no such rule for use of the confession for other purposes.

A dog whistle manufacturer's factory was located near a residential area. The manufacturer used the most effective methods for testing its whistles, but it was impossible to completely soundproof the testing area. A breeder of champion show dogs bought some property near the factory and raised and trained her dogs there. Although the whistles were too high-pitched to be perceived by human ears, they could be heard by the breeder's dogs. Consequently, the dogs often were in a constant state of agitation. In a suit by the breeder against the manufacturer, what is the likely outcome? A. The breeder will prevail on a trespass theory, because the sound waves are entering onto the breeder's property. B. The breeder will prevail on a nuisance theory, because the sound of the whistles is a substantial interference with the breeder's use of her land. C. The breeder will not prevail, because the sound of the whistles is not a substantial interference with the breeder's use of her land. D. The breeder will not prevail, because the manufacturer has acted reasonably in testing its whistles.

C. The breeder will not prevail, because the sound of the whistles is not a substantial interference with the breeder's use of her land. The breeder will not recover because there has been no substantial interference with her use or enjoyment of her land, nor has there been a trespass. A private nuisance is a substantial, unreasonable interference with another person's use or enjoyment of her property. The interference must be offensive, inconvenient, or annoying to the average person in the community. It is not a substantial interference if it merely interferes with a specialized use of the land. Here, the testing of the dog whistles did not bother humans, and so it did not disturb the average person in the community. It is disturbing to the breeder's dogs, but this affects only her specialized use of her land. Thus, the manufacturer's actions do not constitute a private nuisance. (Nor do they constitute a public nuisance—an act that unreasonably interferes with the health, safety, or property rights of the community.) Therefore, (C) is correct, and (B) is incorrect. (A) is incorrect because the sounds reaching the breeder's property do not constitute a trespass. A trespass is an intentional physical invasion of another's land. Sound waves do not produce a physical invasion. Thus, the facts here do not support a basis for trespass. (D) is incorrect because the manufacturer could be found liable to the breeder even if it acted reasonably. In determining whether there is a nuisance, a court would consider the manufacturer's care in testing its whistles, but that factor alone would not be determinative. If the activities were offensive to the average person, the court might still find there is a nuisance—even if it is impossible to do a better job of soundproofing. The court would have to consider the "reasonableness" of the interference, i.e., balance the injury against the utility of the manufacturer's conduct.

The police received a tip from a reliable informant that a former student at the local university was selling narcotics. A brief investigation revealed that the former student, a college dropout, still hung around the university campus, had no visible means of support, and yet drove a large luxury car and wore flashy clothing and jewelry. The police picked up the former student the next time he showed up on campus, took him to the station, and questioned him all night long without a break and without letting him communicate with anyone else. When the former student tired from the interrogation, he admitted that he sold cocaine to his friend, who is a current student at the university. Based on this information, the police went to the current student's dormitory room. When they arrived, they found the door open but no one was in the room. The police entered, searched the room, and discovered a vial of white powder. Later laboratory tests established the powder to be cocaine. The former student was then charged with the sale of narcotics. At his trial, the prosecution attempted to admit the cocaine discovered in the dormitory room into evidence. What is the former student's best argument for preventing the cocaine from being admitted into evidence? A. The search of the dormitory room was conducted without a warrant and without consent. B. The police arrested the former student without a warrant. C. The former student's confession was not voluntary under the circumstances. D. The police failed to give the former student Miranda warnings.

C. The former student's confession was not voluntary under the circumstances. The former student's best argument for preventing the cocaine from being admitted into evidence is that his confession was not voluntary. This question is difficult because each of the choices appears to present a good argument for the former student. With regard to (A), the search of the current student's dorm room appears to be an unreasonable search under the Fourth Amendment. However, a person's Fourth Amendment rights against unreasonable search and seizure may be enforced by the exclusion of evidence only at the instance of someone whose own protection was infringed by the search and seizure. Here, the former student cannot assert a possessory interest or reasonable expectation of privacy in the current student's dorm room. Thus, the former student cannot successfully exclude the cocaine on the ground that it was seized in violation of the Fourth Amendment. (B) is incorrect because arrest warrants are usually required only for arrests made in the person's home. Police generally do not need to obtain a warrant before arresting a person in a public place, even if they have time to get a warrant, as long as the arrest is based on probable cause. Here the police had probable cause to arrest the former student, and because he was arrested on the grounds of the campus, the failure of the police to obtain an arrest warrant will be of no help to him. Choices (C) and (D) both focus on improper conduct during the former student's interrogation, but (C) is better because the former student will have a better chance of invoking the exclusionary rule if the confession is involuntary. For confessions to be admissible, the Due Process Clause of the Fourteenth Amendment requires that they be voluntary. While voluntariness is a fact question that is assessed by looking at the totality of the circumstances, the duration and manner of the police interrogation here indicate that the confession probably was the result of actual coercion. If the confession is found to be involuntary, the former student can invoke the exclusionary rule to exclude the cocaine as "fruit of the poisonous tree." In contrast to an involuntary confession, a confession obtained without Miranda warnings, as long as the failure to warn was not purposeful, may not be sufficient to justify excluding the nontestimonial "fruits" of the confession. [See United States v. Patane (2004)] Thus, the involuntariness of the confession, rather than the absence of Miranda warnings, is the best argument for excluding the cocaine.

A police officer spent several hours using binoculars to observe an older man loitering on a college campus. The man, who was shabbily dressed and carrying a backpack, would approach certain students as they walked by him, and after a brief conversation with them, discreetly pass the students a small envelope in exchange for cash. The officer stopped the man under suspicion that he was dealing drugs. The man was not dealing drugs, but instead had been soliciting donations for a radical political group. The man grew irate when the officer opened one of the envelopes in question and discovered that they only contained literature about the group. The officer then frisked the man and discovered an illegal weapon taped to his leg. The officer immediately arrested the man. Which of the following best describes the situation? A. The officer's actions were unlawful because the officer initially failed to get an arrest warrant before approaching the man, even though he had ample time to do so because the surveillance had been going on for several hours. B. The man's arrest was unlawful because the officer was mistaken about the man selling drugs and thus the weapon would be inadmissible as fruit of the poisonous tree. C. The officer's actions were lawful in stopping the man because the officer had reasonable ground to believe that the man was dealing drugs, but the subsequent search was unlawful once the officer realized his mistake about the drug dealing. D. The stop, search, and subsequent arrest were lawful.

C. The officer's actions were lawful in stopping the man because the officer had reasonable ground to believe that the man was dealing drugs, but the subsequent search was unlawful once the officer realized his mistake about the drug dealing. The stop was lawful but the patdown search was unconstitutional. A police officer has the authority to briefly detain a person for investigative purposes if he has a reasonable suspicion supported by articulable facts of criminal activity. Here, the officer watched the man engage in what reasonably appeared to be drug transactions, and he was justified in stopping and detaining the man to investigate. However, a police officer may pat down a detained person only if the officer has a reasonable suspicion to believe that the detainee is armed and dangerous. Here, the man grew irate when he was stopped, and he belonged to a "radical" group, but these facts are not enough to give the officer any reason to believe that the man was armed. Therefore, the patdown was unconstitutional, and the evidence found as a result must be suppressed. (A) is incorrect. Police generally need not obtain a warrant before arresting a person in a public place, even if they have time to get a warrant. A police officer may arrest a person without a warrant when he has probable cause to believe that a felony has been committed and that the person before him committed it. (B) is incorrect. If during an investigatory detention, the officer develops probable cause for arrest, the officer can proceed on that basis. Although the officer was incorrect in his initial suspicions of drug dealing, the illegal weapon he discovered during the patdown was sufficient to establish probable cause for the man's arrest. (D) is incorrect because, as stated above, an officer may not pat down a detainee for weapons absent a reason to believe the detainee is armed and dangerous.

To secure a loan of $100,000 from a bank, the owner in fee simple of a parcel of land conveyed a deed of trust for the land to the bank. The deed of trust contained a "power of sale" clause, permitted by the jurisdiction, which allowed the bank to sell the property in the event of default without the necessity of a judicial foreclosure action. After several years, the owner defaulted on his loan payments to the bank. The bank informed the owner that it was exercising its power of sale. After appropriate notices, the bank conducted a public sale of the land. The bank was the sole bidder and obtained the property for $80,000, which was $10,000 less than the outstanding balance on the loan plus the expenses of the sale. One month later, the owner notified the bank that he wanted to pay off the loan and extinguish the deed of trust, and was prepared to tender $80,000 to do so. The bank insisted that the owner must tender $90,000 to pay off the loan. If a court in the jurisdiction will require the bank to accept only $80,000 under the circumstances above, what is the likely reason? A. The owner had the power to revoke the trust as long as he was alive. B. The bank did not have the authority to bid on the property at other than a judicial foreclosure sale. C. The owner was exercising a statutory power rather than an equitable power. D. The bank does not have the power to clog the equity of redemption.

C. The owner was exercising a statutory power rather than an equitable power. If the owner can compel the bank to accept his offer, it will be because he has a statutory power to redeem the property after the foreclosure sale has occurred. In all states, the equity of redemption provides the borrower with an equitable right, at any time prior to the foreclosure sale, to redeem the land or free it of the mortgage or lien by paying off the amount due or, if an acceleration clause applies, the full balance due. Only about half the states, however, give the borrower a statutory right to redeem for some fixed period after the foreclosure sale has occurred; the amount to be paid is generally the foreclosure sale price, rather than the amount of the original debt. Thus, if the owner can redeem the land for $80,000, it will be based on the jurisdiction's statutory power of redemption. (A) is wrong because the deed of trust is a security interest (similar to a mortgage) to which the revocation rules for trusts do not apply. The deed of trust was created in part to allow the lender to foreclose on the property without going through a judicial foreclosure proceeding. (B) is wrong because, in states that permit a nonjudicial sale with deeds of trust containing a power of sale, the lender may bid at the sale, and in many cases the lender is the sole bidder. (D) is wrong because the prohibition against "clogging the equity of redemption" refers to the rule that a borrower's right to redeem his own mortgage cannot be waived in the instrument itself. Here, there is nothing to indicate that the owner's deed of trust prohibited him from redeeming the property prior to foreclosure. However, it is only through a statutory right of redemption that the owner would be able to redeem the property for $80,000 after the foreclosure sale had occurred.

A landowner and her neighbor owned adjoining tracts of land. No public road abutted the neighbor's land, so the landowner granted the neighbor an express easement over the north 25 feet of the landowner's land. However, the following month the county extended the public road to the neighbor's land, and he ceased using the easement for ingress and egress. Twenty years later, the neighbor conveyed the easement to his friend, who owned the land adjoining the other side of the landowner. The following year, the neighbor conveyed his land to the landowner. None of the parties has used the easement since the public road was extended. The jurisdiction has a 15-year statute of limitations for acquiring property interests by adverse possession. At what point was the easement extinguished? A. When the neighbor attempted to convey the easement to the friend without conveying the dominant tenement itself. B. Fifteen years after the neighbor ceased using the easement. C. When the neighbor conveyed his land to the landowner. D. The easement was not extinguished

C. When the neighbor conveyed his land to the landowner. The easement was extinguished when the neighbor conveyed his land to the landowner. An easement is extinguished when the easement is conveyed to the owner of the servient tenement. For an easement to exist, the ownership of the easement and the servient tenement must be in different persons. (By definition, an easement is the right to use the land of another for a special purpose.) If ownership of the two property interests comes together in one person, the easement is extinguished. Thus, (D) is wrong. (A) is wrong because, although an attempt to convey an easement appurtenant apart from the dominant tenement is ineffective, it does not extinguish the easement. The easement continues despite the attempted conveyance and will pass with the ownership of the dominant tenement. (B) is wrong because mere nonuse does not extinguish an easement. An easement may be extinguished by abandonment, but to constitute abandonment sufficient to extinguish an easement, the easement holder must demonstrate by physical action an intent to permanently abandon the easement. Nonuse of the easement is not enough to show the intent never to make use of the easement again.

A competitor of a restaurant obtained the restaurant's secret recipe for fried chicken and began using it. The restaurant commenced a diversity action in federal court against the competitor for misappropriation of trade secrets. After the pleadings were served, but before discovery was commenced, the restaurant moved for a preliminary injunction to prevent the competitor from using the recipe. In support of its motion, the restaurant submitted an affidavit of a former employee of the restaurant, who admitted that he stole the recipe from the restaurant and sold it to the competitor. The court granted the restaurant's motion. May the competitor immediately appeal the court's decision? A. No, because the court's decision was based on substantial evidence. B. No, because the grant of a motion for a preliminary injunction is not a final order. C. Yes, because the grant of a preliminary injunction is immediately appealable as of right. D. Yes, because the court should have allowed the competitor to depose the former employee before granting the motion.

C. Yes, because the grant of a preliminary injunction is immediately appealable as of right. (C) The competitor may immediately appeal. A party may appeal as of right any order granting, continuing, modifying, refusing, dissolving, or refusing to dissolve or modify an injunction. (B) is incorrect because, although generally only final orders are appealable, there are exceptions for certain orders, including orders granting injunctions. (A) is incorrect because this choice states a standard of review on appeal, not a basis for an appeal. (D) misconstrues the question. At best, this reason states grounds for asking the appeals court to vacate the injunction. The court's order is appealable because the grant of the injunction is appealable as of right. The grounds for the appeal are not

A developer subdivided a 25-acre tract of land into 100 quarter-acre lots. On each lot she built a two-unit townhouse. The deeds to each of the purchasers contained a covenant that "the grantee, his heirs and assigns" would use the property only for single-family use. All deeds were promptly and properly recorded. Subsequently, the zoning laws were amended to allow multifamily use within the subdivision. Six months later, a social worker offered to purchase an original owner's unit that was for sale. The social worker informed the owner that she planned to operate a halfway house out of the unit, an activity in conformity with the applicable zoning regulations. Therefore, the owner did not include the single-family restriction in the deed to the social worker. If a neighbor, who purchased his lot from the developer, seeks to enjoin the operation of the halfway house, will he succeed? A. No, because the deed from the owner to the social worker did not refer to the covenant. B. No, because the social worker relied on the zoning regulations when purchasing the unit. C. Yes, because the social worker had notice of the restrictive covenant. D. Yes, but only if the neighbor can establish a common scheme for development.

C. Yes, because the social worker had notice of the restrictive covenant The neighbor will succeed in enjoining the operation of the halfway house because the social worker had notice of the restrictive covenant. A covenant runs with the land to a subsequent purchaser with notice of the covenant if it touches and concerns the land and is intended to run. Notice may be actual or constructive. Here, the social worker was on record notice of the covenant because the original owner's deed was recorded. Restricting land to single-family use touches and concerns the land, and it is evident that the developer and the original owners, including the neighbor, intended it to run with the land by use of the language "grantee, his heirs and assigns." The social worker thus will be bound even though her deed did not refer to the covenant. Thus, (A) is incorrect. (B) is incorrect because compliance with zoning regulations does not excuse noncompliance with an enforceable covenant; both must be complied with. (D) is incorrect because the neighbor can prevail without needing to show a servitude implied from a common scheme, which comes into play when a developer subdivides land into several parcels and some of the deeds contain negative covenants and some do not. Here, the covenant relating to single-family use was in all of the original deeds and, as discussed above, it runs with the land. A covenant that runs with the land may be enforced as an equitable servitude if the assignees of the burdened land have notice of the covenant; the usual remedy is an injunction. Here, the social worker had record notice of the covenant and it runs with the land, so the neighbor can enforce the covenant as an equitable servitude without resort to implying a reciprocal negative servitude.

A local news station broadcast a live interview with a bystander about his views concerning the state of local education. The bystander responded by saying that the principal of his daughter's high school had been embezzling school funds for years. The principal saw the telecast and also recorded it. He sued the owner of the station for defamation. At trial, the principal sought to testify to the defamatory statement made in the interview. Will the principal's testimony likely be held to be admissible? A. No, because the testimony would be hearsay not within any exception. B. No, because a recording of the interview exists. C. Yes, because the statement is being offered to show its effect on the principal. D. Yes, because the principal personally saw the interview on television.

D. Because the principal had firsthand knowledge that the statement was made, his testimony will be admissible unless there is a specific rule excluding the evidence. Witnesses are generally presumed competent to testify until the contrary is demonstrated. While a witness may not testify to a matter unless evidence is introduced to support a finding that the witness has personal knowledge of the matter, this evidence may consist of the witness's own testimony. (A) is incorrect. Hearsay is a statement, other than one made by the declarant while testifying at the trial or hearing, offered in evidence to prove the truth of the matter asserted. In a defamation action, evidence of the statement alleged to be defamatory is not hearsay because the evidence is by definition not offered to prove the truth of the matter asserted. It is offered only to show that the actionable statement was made. (B) is incorrect. Because the principal had firsthand knowledge of the event he can testify about the event, even though there might exist a recording that would be better proof of the event. The "best evidence rule" does not apply because the recording is not an essential repository of the facts recorded. (C) is incorrect. Although the statement is not hearsay, it is not being offered to show its effect on the hearer (e.g., knowledge, motive), but rather to show that the statement was made, as explained above.

A town in a rural state facing financial difficulties passed a variety of "sin taxes," including one aimed at electronic game arcades frequented by local juveniles. The tax is a one cent per game tax imposed on the manufacturers of the games based on the estimated number of plays over a machine's lifetime. There are no electronic game manufacturers in the state. Which of the following constitutional provisions would support the best argument against enforcement of the tax? A. The Equal Protection Clause. B. Substantive due process. C. The Privileges and Immunities Clause of Article IV. D. The Commerce Clause.

D. The best argument against enforcement of the tax is that it violates the Commerce Clause. If Congress has not adopted laws regarding a subject, local governments are free to tax or regulate local aspects of the subject area as long as the tax or regulation does not discriminate against interstate commerce or unduly burden it. Here, the tax does not discriminate against interstate commerce, since it does not single out interstate commerce for taxation in order to benefit the local economy. However, it could be argued that the tax unduly burdens interstate commerce. A local tax will be held to unduly burden interstate commerce if the locality's need for the revenue does not outweigh the burden on interstate commerce. The Supreme Court will consider whether there is a substantial nexus between the activity or property taxed and the taxing state, whether the tax is fairly apportioned, and whether there is a fair relationship between the tax and the benefit the taxed party receives from the state. Here, there is little nexus between the manufacturer and the town. The facts indicate that out-of-state manufacturers' machines are used in the town, but do not indicate whether the manufacturers conduct any selling activity in the town. Similarly, nothing indicates that there is a relationship between the tax and any benefit that the manufacturers derive from the town. Thus, the tax would probably be unconstitutional under the Commerce Clause. (A) is not as good an argument as (D) because the Equal Protection Clause prohibits the states from treating similarly situated persons differently without sufficient justification. Where a classification does not involve a suspect or quasi-suspect class or a fundamental right, the classification will be upheld as long as it is rationally related to a legitimate government interest. While the tax here singles out arcade game manufacturers for special tax treatment, no suspect or quasi-suspect class is involved, nor is a fundamental right affected. Thus, the tax will be valid under the Equal Protection Clause because it is rationally related to the legitimate government interest of raising revenue. (B) is not a good argument because substantive due process requires that laws not be arbitrary. When laws do not involve a fundamental right, they will be held valid under the Due Process Clause as long as they are rationally related to a legitimate government interest. As established above, no fundamental right is involved and the tax is rationally related to a legitimate government interest. Thus, under the Due Process Clause the tax may be enforced. (C) is not a good argument because the Privileges and Immunities Clause of Article IV prohibits states from discriminating against out-of-state residents when a fundamental right is involved, and the tax here does not differentiate between residents and nonresidents.

On February 1, the owner of a bowling alley read in a magazine an ad from a major manufacturer of bowling balls offering sets of 40 balls in various weights and drilled in various sizes for $10 per ball. The owner immediately filled out the order form included in the ad for the 40 balls and deposited it, properly stamped and addressed, into the mail. On February 2, the bowling alley owner received in the mail a letter from the manufacturer, sent out as part of its advertising campaign, stating in relevant part that it will sell the bowling alley owner 40 bowling balls at $10 per ball. A day later, on February 3, the manufacturer received the bowling alley owner's order. On February 4, the balls were shipped. On what day did an enforceable contract arise? A. February 1, the day the bowling alley owner deposited his order in the mail. B. February 2, the day the bowling alley owner received the letter from the manufacturer. C. February 3, the day the manufacturer received the bowling alley owner's letter. D. February 4, the day the balls were shipped.

D. The contract arose when the balls were shipped. The general rule is that an offer can be accepted by performance or a promise to perform unless the offer clearly limits the method of acceptance. Here, the offer would be the bowling alley owner's order, because a magazine ad is usually held to be merely solicitation to accept offers rather than an offer. Thus, the manufacturer accepted and the contract was formed when it shipped the balls. (A) is wrong because the bowling alley owner's order was an offer to buy, and no contract could be formed until that offer was accepted. (B) is wrong because this is a case of crossing offers; even though both offers contain the same terms, they do not form a contract. (C) is wrong because no contract will be formed until there has been an acceptance, and, as stated, the bowling alley owner's letter was merely an offer.

Juxtapose this question with the above question. In this question, the offer is not an "ad." On December 6, the owner of an electronics store sent a written request to a computer manufacturer asking for the price of a certain laptop computer. The manufacturer sent a written reply with a catalog listing the prices and descriptions of all of his available computers. The letter stated that the terms of sale were cash within 30 days of delivery. On December 14, by return letter, the store owner ordered the computer, enclosing a check for $4,000, the listed price. Immediately on receipt of the order and check, the manufacturer informed the store owner that there had been a pricing mistake in the catalog, which should have quoted the price as $4,300 for that computer. The store owner refused to pay the additional $300, arguing that his order of December 14 in which the $4,000 check was enclosed was a proper acceptance of the manufacturer's offer. In a suit for damages, will the manufacturer prevail? A. Yes, because his first communication stated terms calling for cash within 30 days of delivery. B. Yes, because of the mistake as to price. C. Yes, because his first communication did not constitute an offer. D. No, because the store owner's December 14 letter was a proper acceptance of the manufacturer's offer.

D. The store owner's December 14 letter was an acceptance. Whether the letter was an acceptance depends on whether the manufacturer's letter was an offer, because an acceptance is a manifestation of assent to an offer. For a communication to be an offer, it must create a reasonable expectation in the offeree that the offeror is willing to enter into a contract on the basis of the offered terms. There must be a promise, undertaking, or commitment to enter into a contract with certain and definite terms. Courts usually hold that if a statement is made broadly, such as in an advertisement or catalog, it will not constitute an offer because it is not reasonable to expect that the sender intended to make offers to all who received the advertisement; rather, the courts usually find such advertisements to be invitations seeking offers. However, price quotations may be considered as offers if given in response to a specific inquiry. The courts will look to the surrounding circumstances, and here a court would probably determine that the catalog that the manufacturer sent was an offer because it was sent in response to the store owner's specific inquiries about prices on a specific computer and it included delivery terms and conditions of sale. (A) is incorrect because although the letter called for payment in cash, tender by check is sufficient unless the seller demands legal tender and gives the buyer time to obtain cash. Moreover, because the contract called for payment within 30 days of delivery, even if the check was not sufficient, the store owner still had time under the contract to obtain cash. (B) is incorrect because the mistake was unilateral. Generally, a unilateral mistake will not be grounds to rescind a contract unless the nonmistaken party knew or should have known of the mistake. Here, nothing in the facts indicates that the store owner knew of the mistake, and the mistake was not so large that it could be said that he should have known of it. (C) is incorrect because, as explained above, the manufacturer's catalog was sent in response to the store owner's request for information and his terms for sale constituted an offer.

On December 6, the owner of an electronics store sent a written request to a computer manufacturer asking for the price of a certain laptop computer. The manufacturer sent a written reply with a catalog listing the prices and descriptions of all of his available computers. The letter stated that the terms of sale were cash within 30 days of delivery. On December 14, by return letter, the store owner ordered the computer, enclosing a check for $4,000, the listed price. Immediately on receipt of the order and check, the manufacturer informed the store owner that there had been a pricing mistake in the catalog, which should have quoted the price as $4,300 for that computer. The store owner refused to pay the additional $300, arguing that his order of December 14 in which the $4,000 check was enclosed was a proper acceptance of the manufacturer's offer. In a suit for damages, will the manufacturer prevail? A. Yes, because his first communication stated terms calling for cash within 30 days of delivery. B. Yes, because of the mistake as to price. C. Yes, because his first communication did not constitute an offer. D. No, because the store owner's December 14 letter was a proper acceptance of the manufacturer's offer.

D. The store owner's December 14 letter was an acceptance. Whether the letter was an acceptance depends on whether the manufacturer's letter was an offer, because an acceptance is a manifestation of assent to an offer. For a communication to be an offer, it must create a reasonable expectation in the offeree that the offeror is willing to enter into a contract on the basis of the offered terms. There must be a promise, undertaking, or commitment to enter into a contract with certain and definite terms. Courts usually hold that if a statement is made broadly, such as in an advertisement or catalog, it will not constitute an offer because it is not reasonable to expect that the sender intended to make offers to all who received the advertisement; rather, the courts usually find such advertisements to be invitations seeking offers. However, price quotations may be considered as offers if given in response to a specific inquiry. The courts will look to the surrounding circumstances, and here a court would probably determine that the catalog that the manufacturer sent was an offer because it was sent in response to the store owner's specific inquiries about prices on a specific computer and it included delivery terms and conditions of sale. (A) is incorrect because although the letter called for payment in cash, tender by check is sufficient unless the seller demands legal tender and gives the buyer time to obtain cash. Moreover, because the contract called for payment within 30 days of delivery, even if the check was not sufficient, the store owner still had time under the contract to obtain cash. (B) is incorrect because the mistake was unilateral. Generally, a unilateral mistake will not be grounds to rescind a contract unless the nonmistaken party knew or should have known of the mistake. Here, nothing in the facts indicates that the store owner knew of the mistake, and the mistake was not so large that it could be said that he should have known of it. (C) is incorrect because, as explained above, the manufacturer's catalog was sent in response to the store owner's request for information and his terms for sale constituted an offer.

A state adopted a law requiring that all clothing sold within the state be treated with fire-retardant material. The law empowered a state agency to issue rules specifying the required treatment. A manufacturer of clothing that sold its products in the state brought suit in federal court for a declaratory judgment as soon as the law was adopted, alleging that the statute violated the United States Constitution.What should the court do? A)Entertain the suit, because a federal question is involved. B) Entertain the suit, because an action for declaratory relief is a proper method of deciding constitutional questions. C)Abstain from the assertion of jurisdiction until a state court has the opportunity to construe the statute. D)Dismiss the suit.

D. The statute imposes a requirement but regulations have to be developed to implement it. Until those regulations are adopted, it cannot be known what's prohibited and what's allowed. so, at this stage, the matter is not ripe for review.

A homeowner entered into a written agreement with a contractor whereby the contractor agreed to completely remodel the homeowner's bathroom "to her specifications" at a cost of $10,000. The homeowner's specifications were highly detailed and required custom-made fixtures that would not be usable in other bathroom remodeling jobs. The contractor ordered the custom-made fixtures and paid $4,000 for them when they were delivered to his place of business. Figuring up the cost of the fixtures and labor, the contractor estimated that he would make a total profit of $2,000 on the job after payment for materials and workers. Before the contractor began work on the project, but after he had paid for the fixtures, the homeowner told the contractor that she had had a change of heart and would probably be selling the house the following year, and so would not need a custom bathroom. The contractor made no attempt to sell the fixtures to another contractor and filed suit against the homeowner for damages. What is the contractor likely to recover? A. Nothing, because he failed to mitigate damages. B. His expectation damages of $2,000. C. $4,000, the cost of materials as restitution. D. $2,000 as expectation damages, plus $4,000 in reliance damages.

D. $2,000 as expectation damages, plus $4,000 in reliance damages. The contractor can recover $2,000 as lost profits plus the $4,000 in costs he incurred before the homeowner breached the contract. The purpose of a damages remedy based on an affirmance of the contract is to give compensation for the breach; i.e., to put the nonbreaching party where he would have been had the promise been performed. In most cases, the plaintiff's standard measure of damages will be based solely on an "expectation" measure, i.e., sufficient damages for him to buy a substitute performance. A reliance measure of damages, on the other hand, awards the plaintiff the cost of his performance, i.e., his expenditures in performing his duties under the contract. In certain situations, an award of compensatory damages will contain both an expectation and a reliance component. In a construction contract, if the owner breaches the contract after the builder has already begun his performance, the builder will be entitled to any profit he would have derived from the contract plus any costs he has incurred to date. This formula contains an expectation component (the profit the builder would have made) and a reliance component (the cost incurred prior to the breach). This formula is applicable to the facts in this case. The contractor has begun performance by ordering and purchasing the custom-made fixtures at a cost of $4,000. Because they are usable only for the homeowner's purposes, their cost, which is treated just like any other expenditure of labor and material in a partially completed construction contract, can be recovered as reliance damages. The other element of his recovery is the $2,000 profit that he would have derived from the contract—his expectation damages. His total recovery will therefore be $6,000. (A) is incorrect because the contractor can do nothing further to mitigate his damages. The nonbreaching party is always under a duty to mitigate damages after learning of the other party's breach. In construction contracts, the builder's duty to mitigate generally dictates only that he not continue work after the breach and not incur further expenditures. While the builder would also have a duty to apply any usable materials that he purchased to other jobs or to attempt to resell them to another contractor, the facts specify that the custom-made fixtures here were not usable in other remodeling jobs. Hence, the contractor's failure to attempt to sell the fixtures did not amount to a failure to mitigate damages. (B) is incorrect because an award of $2,000 does not put the contractor in the position he would have been in had the contract been performed—the $4,000 that he spent on the fixtures would have been covered by part of the $10,000 that he was to receive as the contract price. Had the homeowner not breached, the contractor would have received the contract price of $10,000, and he would have spent an additional $4,000 in labor and materials to complete the job. The difference, $6,000, consists of the $4,000 that he already spent on materials and the $2,000 profit that he expected to make. (C) is incorrect. Instead of seeking a damages remedy based on an affirmance of the contract, the nonbreaching party may rescind and sue for restitution for any "benefit" that he has transferred to the breacher in an attempt to perform the contract. The restitution recovery is generally based on the fair market value of the benefit transferred. Here, the contractor can provide the homeowner with the fixtures and seek restitution, but there is nothing to indicate that their fair market value is $4,000. Even if that is the case, however, the contractor has a provable compensatory damages remedy on the contract of $6,000; he will elect that remedy rather than the lesser restitution remedy.

Which of the following is not considered a type of "fighting words" that may be forbidden by statute? A. Statements likely to incite physical retaliation. B. Conduct undertaken with the intent to cause fear of bodily harm (e.g., cross burning). C. Statements meant to place a person in fear of bodily harm. D. "Abusive language."

D. Abusive language

To increase tourism, a city began sponsoring laser light shows, which proved to be very popular. Several charitable organizations received permission from the council to sponsor a show and charge admission to raise money to help support their causes. One of them hired a famous laser light artist to give their show. When the artist arrived, he began setting up his lasers for the show. A city official soon stopped him, informing him that he could use only the city's lasers because the city feared that outsiders might use powerful lasers that could cause eye damage to viewers. The artist told the charitable organization that had hired him that the success of his art depends on the power of his lasers and that he could not produce desirable effects using the city's lasers. The charitable organization appealed to the city, but the city held fast to its rule requiring all laser light artists to use the city's lasers. If the charitable organization files an action against the city, how will the court most likely rule? A. Find for the charitable organization, because art is protected by the First Amendment and the city rule interferes with the artist's freedom of expression. B. Find for the charitable organization, because the city rule is not the least restrictive method for achieving the city's goals. C. Find for the city, because the laser light show is not speech and therefore is not protected by the First Amendment. D. Find for the city, because the rule is a reasonable time, place, and manner restriction.

D. Find for the city, because the rule is a reasonable time, place, and manner restriction. The city will prevail because its rule is a reasonable time, place, and manner restriction. Speech protected by the First Amendment includes not only verbal communication, but also conduct that is undertaken to communicate an idea. The laser light show, like other art, probably is protected speech. While the content of speech generally cannot be limited, the conduct associated with speech in public forums can be regulated by reasonable time, place, and manner restrictions. To avoid strict scrutiny and be upheld, such a regulation must be content neutral, narrowly tailored to serve an important government interest, and leave open alternative channels of communication. The city's rule meets these requirements: The types of images displayed are not controlled, just the means of showing them; the rule is narrowly tailored because it does not regulate substantially more speech than is necessary to further an important government interest (here, preventing eye damage); and alternative channels of communication are available because the artist can use the city's equipment, albeit with less spectacular results. (A) is incorrect because while the artist's art is protected by the First Amendment, it may still be regulated by reasonable time, place, and manner regulations, as indicated above. (B) is incorrect because it states the wrong standard. A time, place, and manner regulation need not be the least restrictive means for achieving the desired result, but rather only narrowly tailored to the result. [See Ward v. Rock Against Racism (1989)] (C) is incorrect because art, including performance art such as the laser light show, is protected by the First Amendment. As discussed above, the First Amendment guarantee of freedom of speech protects more than merely spoken or written words; it includes conduct and other forms of expression undertaken to communicate an idea.

A new federal law prohibited the use of various pesticides in areas with a certain population density near navigable waters. A city located in the southeastern United States was plagued by a sharp increase in disease-carrying mosquitoes. The city's board of health recommended that all residential areas be sprayed with a pesticide proven to be highly effective against mosquitoes. Despite the fact that the federal law would prohibit use of that pesticide in these areas, the city council passed an ordinance adopting the board of health plan, relying on the opinions of several independent experts that the health benefits of reducing the mosquito population outweighed the risks of spraying. An environmentally minded citizen of the city brought an action in federal court challenging the ordinance. Assuming that the citizen has standing, is the court likely to find the ordinance valid? A. Yes, because pursuant to the police power, cities have a compelling interest in laws designed to protect the health, safety, and welfare of their citizens. B. Yes, because controlling health hazards is an integral governmental function. Incorrect C. No, because it is superseded by the power of Congress to adopt laws to protect the health, safety, and welfare of citizens. D. No, because it conflicts with a federal law that Congress had the power to make under the Commerce Clause.

D. No, because it conflicts with a federal law that Congress had the power to make under the Commerce Clause. Congress's power to regulate commerce has been construed broadly, so that it may regulate any activity, local or interstate, that either in itself or in combination with other activities has a substantial economic effect on interstate commerce. If Congress has determined that the use of chemical pesticides and their runoff into waterways (which are channels of interstate commerce) will have an overall detrimental impact on the environment, this determination will be sufficient in this case to satisfy the standards established by the Supreme Court. Therefore, the law probably is a valid exercise of the commerce power. Any state or local action that conflicts with a valid act of Congress is invalid under the Supremacy Clause. (A) is incorrect because while the police power (the power to adopt regulations for the health, safety, and welfare of citizens) belongs to the states, a police power regulation that conflicts with a federal law is invalid under the Supremacy Clause. (B) is incorrect because state and local government activities may be regulated by a general law that applies to both the public and private sectors, even if the regulation affects integral governmental functions, as long as there is a constitutional basis for the law. (C) is incorrect because Congress does not have a general "police power" to adopt laws on health and safety. The laws that Congress has passed banning activities that it has deemed harmful to public health have been based on its power to regulate interstate commerce.

A homeowner and a local builder entered into a written contract that called for the builder to build a second story onto the top of the homeowner's one-story residence. When scheduling conflicts arose, the builder asked the homeowner if they could substitute his buddy, an out-of-town builder who had comparable experience and skills, to perform the local builder's part of the contract. All of the parties agreed to the substitution. Unfortunately, the out-of-town builder made a major blunder that will be quite expensive to correct. Is the local builder liable to the homeowner for the cost of correcting the defect? A. Yes, because the substitution in and of itself does not relieve the local builder of liability on the underlying contract. B. Yes, because the local builder did not give any consideration on which to base a release. C. No, because the local builder transferred his duties to the out-of-town builder. D. No, because the local builder was discharged through a novation.

D. No, because the local builder was discharged through a novation. The agreement among all of the parties to substitute the out-of-town builder for the local builder operates as a novation which immediately discharged the local builder from any duties he had under the original contract. A novation arises when the parties enter into an agreement to substitute a third party for one of the parties in a contract, releasing the party who was substituted. All parties must agree to the substitution. Here, the facts say that all of the parties agreed that the out-of-town builder would substitute for the local builder. Thus, there was a novation and the local builder was released immediately and is not liable for the out-of-town builder's blunder. (A) states incorrectly that the substitution does not relieve the local builder of liability. If the parties had not all agreed to substitute the out-of-town builder for the local builder, or the facts said that there was merely an assignment of rights and delegation of duties, (A) would reflect the correct result. However, where the parties agree to substitute a new party for an old party, there is a novation that does release the old party. (B) is incorrect because there was consideration to support the release—the local builder implicitly agreed to give up his rights under the original contract, the homeowner implicitly agreed to give up his right to look to the local builder for performance, and the out-of-town builder agreed to perform. (C) is not as good an answer as (D). The mere fact that a contractual duty was transferred does not release the transferor from a duty under the contract. It is only the agreement among the parties to substitute the new party for the old that released the local builder here.

In a criminal battery case brought against the defendant, the prosecutor asked the court to take judicial notice of the fact that a car driven from Chicago to Detroit has to cross state lines. The defense attorney raised no objection, and the judge declared that she was taking judicial notice of the fact as requested by the prosecution. What is the effect of such judicial notice? A. To raise an irrebuttable presumption. B. To satisfy the prosecutor's burden of persuasion on that issue. C. To shift the burden of persuasion on that issue to the defendant. D. That the judge should instruct the jury that it may, but is not required to, accept the noticed fact as conclusively proven.

D. That the judge should instruct the jury that it may, but is not required to, accept the noticed fact as conclusively proven. The effect of the judge's noticing that a car driven from Chicago to Detroit must cross state lines is that the judge will now instruct the jury that it may, but is not required to, accept that fact as conclusively proven. Under the Federal Rules, in a civil case, the court must instruct the jury to accept the judicially noticed fact as conclusive. [Fed. R. Evid. 201(f)] Because this question deals with a prosecution for criminal battery, the applicable rule is that the jury be instructed that the fact that has been judicially noticed may be accepted by it as conclusive, but that the jury is not required to do so. (A) would be correct if this were a civil case. In such an instance, the jury would be instructed to accept as conclusive the judicially noticed fact. This would have the effect of raising an irrebuttable presumption. (B) is incorrect because, in a criminal case, the prosecution has the burden of proving every element of the crime beyond a reasonable doubt. Only the jury can decide, after all of the evidence is in, whether the burden of persuasion is satisfied. (C) is incorrect because the burden of persuasion does not shift from party to party during the course of the trial. The burden of persuasion is never on a criminal defendant.

A manufacturer of widgets was incorporated in and has its principal place of business in State A. The manufacturer also operates its own stores in State A, State B, and State C, through which it sells its widgets to consumers. A consumer who resides in State D visited a State D store to purchase a widget. The State D store did not have the widget he needed, and employees at the store directed him to the State B store operated by the manufacturer. The consumer visited the manufacturer's State B store and purchased the widget. The consumer returned to his home in State D, where he was injured while using the widget. The consumer intends to file a products liability action against the manufacturer in federal district court. In what district or districts is venue proper? A. The District of State A, the District of State B, and the District of State C. B. The District of State A only. C. The District of State A and the District of State D only. D. The District of State A, the District of State D, and the District of State B.

D. The District of State A, the District of State D, and the District of State B. Venue is proper in the District of State A, the District of State D, and the District of State B. Federal venue in civil actions is proper in (i) the district where any defendant resides, if all defendants are residents of the state in which the district is located; and (ii) the district in which a substantial part of the events or omissions giving rise to the claim occurred. The District of State D is proper because a substantial part of the events giving rise to the claim—use of the widget and the injury—took place there. Substantial events also occurred in the District of State B (the sale). Additionally, venue is proper in other districts because the manufacturer is deemed to reside there. Corporations are deemed to reside in any district in which it is subject to personal jurisdiction with respect to the action in question. The manufacturer is subject to personal jurisdiction and thus deemed to reside in the District of State A (because its incorporation and principal place of business are there, thus subjecting it to general jurisdiction there), the District of State B (because it sold the widget from its store there, thus subjecting it to specific jurisdiction there with respect to the current action), and the District of State D (because the injury took place in State D, thus subjecting it to specific jurisdiction there with respect to the current action). Thus, (B) and (C) are incorrect. (A) is incorrect because, although the manufacturer transacted business in the District of State C (because it operates stores there), such activity would not subject the manufacturer to personal jurisdiction with respect to the action here, because State C has no connection with the consumer's injury. As a result, the manufacturer does not reside, for the current action, in that district for venue purposes.

A defendant charged with driving while intoxicated pleaded not guilty and insisted on a trial. Right before the trial began, he fired his attorney and decided to defend himself. At one point during opening arguments, the defendant began to act like a cat, meowing and chasing an imaginary squirrel out of the courtroom. If no one else raises the issue of the defendant's competency to stand trial, what is the responsibility of the trial judge here? A. The trial judge has no responsibility, because the defendant decided to defend himself. B. The trial judge has no responsibility, because she cannot decide whether the defendant is competent to stand trial. C. The trial judge must raise the issue of competency, because the defendant is representing himself. D. The trial judge must raise the issue of competency, because the Constitution obligates her to do so.

D. The trial judge must raise the issue of competency, because the Constitution obligates her to do so. The judge must raise the issue of competency. If it appears to the judge that the defendant might be incompetent, the judge has a constitutional obligation to conduct further inquiry and determine whether in fact the defendant is incompetent. If the defendant is tried and convicted but it later appears that he was incompetent to stand trial, the judge's failure to raise the issue or to request a determination of competency does not constitute a waiver of the competency issue. [Pate v. Robinson (1966)] Therefore, if the trial judge observes the defendant acting in such a way that may indicate he is incompetent to stand trial (e.g., meowing, chasing imaginary squirrels), she should conduct further inquiry to determine the competency of the defendant. (A) is incorrect. As explained above, the trial judge has a duty to conduct an independent inquiry if there is evidence that the defendant may not be competent to stand trial. This would be true even if the defendant were represented by counsel. (B) is incorrect. As explained above, the judge can and should decide the issue of competency if there is evidence that the defendant might not be competent to stand trial. (C) is incorrect. If there is evidence that the defendant might not be competent to stand trial, the judge should conduct an independent inquiry into competency regardless of whether the defendant is representing himself or is represented by an attorney.

Small, prolific mussels called zebra mussels were first introduced into the Great Lakes by a foreign cargo ship. They became a serious problem because they attached themselves to smooth, hard surfaces, and often clogged water intake pipes. Congress determined that zebra mussels posed a great threat to the economic welfare of the Great Lakes region and passed a statute requiring all Great Lakes water intakes to be coated with a special chemical compound that repels zebra mussels. Studies by biologists at a major state university showed that while the special chemical compound that the federal government has required was effective, it also was toxic to other aquatic life. The biologists recommended that Great Lakes intake pipes be coated with a less toxic and less expensive copper-based paint. On the basis of those studies and the recommendation, three Great Lakes states adopted laws permitting municipal water districts to coat their intake pipes with copper paint. Can municipalities using copper-based paint on their intake pipes successfully be prosecuted for violating the federal law? A. No, because the Tenth Amendment prevents Congress from interfering with integral government functions. B. No, because the municipalities are taking effective steps to combat zebra mussels in compliance with the spirit and purpose of the federal law. C. Yes, because Congress is in a better position to regulate the entire Great Lakes region than the individual states. D. Yes, because Congress may adopt laws regulating navigable waters.

D. Yes, because Congress may adopt laws regulating navigable waters. The cities can be prosecuted because state or local government action that conflicts with valid federal laws is invalid under the Supremacy Clause. The federal law here could be found valid as an exercise of the commerce power (Congress can regulate any activity that either in itself or in combination with other activities has a substantial economic effect on interstate commerce) or under the admiralty power (Congress can regulate all navigable waterways). The action of the municipalities directly conflicts with the directives of the federal law and can therefore be stopped. (B) is incorrect because the fact that the copper paint may be as effective as the special compound does not change the result. The action by the municipalities can be prohibited under the Supremacy Clause. (A) is incorrect because for regulations that apply to both the public sector and the private sector, the Supreme Court has held that states' Tenth Amendment rights are best protected by the states' representation in Congress; hence, the Tenth Amendment is not a likely ground for striking this federal legislation because it is not directed only at state or local governments. (C) is incorrect because it is irrelevant; the federal law is superior to the states' laws because it is within Congress's power, not because Congress is in a better position than the states to adopt the legislation involved.

A State A citizen was arrested by a police officer in State A. The State A citizen filed a civil action against the police officer in a State A state court. The complaint alleges that the police officer wrongly beat the citizen in the course of the arrest and seeks money damages under both state tort law and under 42 U.S.C. section 1983 for violation of the citizen's civil rights. The police officer, who is also a citizen of State A, promptly filed a notice of removal to federal court. Is the case properly removable? A. No, because the citizen and police officer are citizens of the same state. B. No, because the police officer is a citizen of State A, the state in whose court the case is pending. C. Yes, as long as the amount in controversy exceeds $75,000. D. Yes, because one of the claims arises under federal law and the federal court has supplemental jurisdiction over the other claim.

D. Yes, because one of the claims arises under federal law and the federal court has supplemental jurisdiction over the other claim. The case is properly removable. Under 28 U.S.C. section 1441, a defendant may remove an action that could have originally been brought in the federal courts. Federal question jurisdiction is available when the plaintiff, in his well-pleaded complaint, alleges a claim that arises under federal law. In the instant case, the citizen's civil rights claims under section 1983 present a federal question. Because a federal question has been presented, the case is removable. Once one claim satisfies the requirements for original federal subject matter jurisdiction, the court has discretion to exercise supplemental jurisdiction over related claims that derive from the same common nucleus of fact and are such that a plaintiff would ordinarily be expected to try them in a single judicial proceeding. Here, the citizen's state tort law claim is derived from the exact same fact pattern. Thus, the court has supplemental jurisdiction over that claim. As a result, (D) is correct. When removal is based on federal question jurisdiction, unlike diversity jurisdiction, there is no amount in controversy requirement, nor is there any requirement that the parties be from different states. Thus, (A) and (C) are incorrect. (B) is incorrect because the "in-state defendant" restriction on removal applies only when removal is based solely on diversity of citizenship jurisdiction.

A landowner leased 150 acres of farmland to a produce company for 15 years. The produce company used the land for crops along with several other contiguous acres that it owned or leased. About four years into the lease, the state condemned a portion of the leased property because it intended to build a highway. As a result, too little property remained for the produce company to profitably farm, although there still existed the farmhouse on the property, which was being used by one of its foremen. The produce company gave the landowner 30 days' written notice that it considered the lease to have been terminated because of the condemnation. In a suit for breach of contract, is the landowner likely to win? A. No, because the condemnation made it economically undesirable for the produce company to continue to lease the property. B. No, because when there is a condemnation, the tenant's obligation to pay rent is extinguished. C. Yes, because the produce company can still use the farmhouse, and the rental value would be adjusted accordingly. D. Yes, because the condemnation did not affect the produce company's obligation to pay the full rental price, although it is entitled to share in the condemnation award.

D. Yes, because the condemnation did not affect the produce company's obligation to pay the full rental price, although it is entitled to share in the condemnation award. The landowner probably will win in a breach of contract suit. In partial condemnation cases, the landlord-tenant relationship continues, as does the tenant's obligation to pay the entire rent for the remaining period of the lease. The tenant is, however, entitled to share in the condemnation award to the extent that the condemnation affected the tenant's rights under the lease. Therefore, (B) and (C) are incorrect. (A) is not correct because the law of landlord and tenant traditionally refuses to recognize frustration of purpose as grounds for termination of a lease.

A resident of City A (located in the Northern District of State A) brought a diversity action against a resident of City B (located in the Eastern District of State B). The cause of action arose in City B. The defendant commutes daily from his home in City B to his office in City A and would find it much more convenient to litigate the action there than in City C, where the appropriate court for the Eastern District of State B is located. The plaintiff would also find it more convenient to litigate in City A, but will go to City C if necessary. May the action be litigated in City A? A. No, because venue is not proper there. B. No, if process is served in State A. C. Yes, because a plaintiff is entitled to choose venue. D. Yes, if the defendant waives proper venue.

D. Yes, if the defendant waives proper venue The action may be litigated in City A because improper venue may be waived. The defendant may inform the plaintiff that he will waive any objection to venue in City A. Given that the plaintiff has indicated a willingness to bring the action in City C if necessary, the defendant would not seem to gain any advantage by insisting on proper venue at great inconvenience to him, unless other factors make City C a preferable venue. (A) is incorrect because although City A is not a proper venue under 28 U.S.C. section 1391, improper venue may be waived. (B) is incorrect because the place where process is served has no effect on venue. (C) is incorrect because a defendant may object to improper venue.

cross-examination is generally limited in scope to the subject matter of the direct examination and ...?

Matters affecting the credibility of the witness

A landowner possessed a 40-acre tract of land. He had inherited 30 acres and had possessed the other 10 acres for longer than the statutory period necessary to acquire title by adverse possession from a rancher. The landowner entered into a land sale contract promising to convey the 40 acres to a developer. The contract provided that the landowner would convey marketable title. The developer paid the landowner the purchase price and accepted a deed from him. The developer promptly recorded the deed. The rancher, having learned of the sale, brought a successful action against the developer to quiet title. The developer realized for the first time that there were no covenants for title in his deed. The developer brings an action against the landowner. What is the most likely outcome of the suit? A. The developer will win, because the landowner breached the terms of the contract. B. The developer will win, because the landowner misrepresented the size of the tract. C. The landowner will win, because the terms of the deed control his liability. D. The landowner will win, because the developer was negligent in not checking the covenants of title at the time of closing.

The landowner will win because the terms of the deed, not of the contract, control his liability. There is an implied covenant in every land sale contract that at closing the seller will provide the buyer with a title that is "marketable." Marketable title is title reasonably free from doubt, i.e., title that a reasonably prudent buyer would be willing to accept. It need not be a "perfect" title, but the title must be free from questions that might present an unreasonable risk of litigation. Generally, this means an unencumbered fee simple with good record title. Generally, a title acquired by adverse possession is not considered marketable because the purchaser might be later forced to defend in court the facts that gave rise to the adverse possession against the record owner. Here, the marketability requirement did not have to be implied, it was an express term of the contract. Under the doctrine of merger, the contract merges into the deed, and the terms of the contract are meaningless. Even though the contract specified a "good and marketable title," it is the deed that controls, and the deed contained no covenants of title. A deed does not incorporate the title terms of a contract. Thus, (A) is wrong. (B) is wrong; it is not supported by the facts. (D) is wrong because the developer's negligence is irrelevant.

Detrimental reliance applies when...?

There is no contract. the contract is lacking consideration.


Conjuntos de estudio relacionados

N1 Unit 6 Isolation & Asepsis/Infection

View Set

A+ Ch. 7 Questions, A+ Ch. 8 Questions, A+ Ch. 9 Questions, A+ Ch. 10 Questions

View Set